+ All Categories
Home > Documents > SIMPLYFYING IAS EXAM PREPARATION...• He also founded the organisation Shri Mathura Vrindavan...

SIMPLYFYING IAS EXAM PREPARATION...• He also founded the organisation Shri Mathura Vrindavan...

Date post: 06-Sep-2020
Category:
Upload: others
View: 2 times
Download: 0 times
Share this document with a friend
91
INSIGHTSIAS SIMPLYFYING IAS EXAM PREPARATION INSTA Tests 33 to 36 (GS) www.insightsonindia.com prelims.insightsonindia.com | mains.insightsonindia.com Telegram: insightsIAStips | FB: insightsonindia | TW: vinaygb | YT: INSIGHTS IAS BENGALURU | DELHI | HYDERABAD INSTA 75 Days REVISION PLAN UPSC Prelims 2020 Copyright © by Insights IAS All rights are reserved. No part of this document may be reproduced, stored in a retrieval system or transmitted in any form or by any means, electronic, mechanical, photocopying, recording or otherwise, without prior permission of Insights IAS. KEY & EXPLANATIONS
Transcript
Page 1: SIMPLYFYING IAS EXAM PREPARATION...• He also founded the organisation Shri Mathura Vrindavan Hasanand Gochar Bhoomi in Vrindavan. 5. Periyar E. V. Ramasamy was associated with which

INSIGHTSIAS SIMPLYFYING IAS EXAM PREPARATION

INSTA Tests

33 to 36 (GS)

www.insightsonindia.com

prelims.insightsonindia.com | mains.insightsonindia.com

Telegram: insightsIAStips | FB: insightsonindia | TW: vinaygb | YT: INSIGHTS IAS

BENGALURU | DELHI | HYDERABAD

INSTA 75 Days REVISION PLAN UPSC Prelims 2020

Copyright © by Insights IAS All rights are reserved. No part of this document may be reproduced, stored in a retrieval system or transmitted in any form or by any means, electronic, mechanical, photocopying, recording or otherwise, without prior permission of Insights IAS.

KEY & EXPLANATIONS

Page 2: SIMPLYFYING IAS EXAM PREPARATION...• He also founded the organisation Shri Mathura Vrindavan Hasanand Gochar Bhoomi in Vrindavan. 5. Periyar E. V. Ramasamy was associated with which

Insta 75 Days Revision Plan for UPSC Civil Services

Prelims – 2020

This document is the compilation of 100 questions that are part of InsightsIAS

famous INSTA REVISION initiative for UPSC civil services Preliminary examination

– 2020 (which has become most anticipated annual affair by lakhs of IAS aspirants

across the country). These questions are carefully framed so as to give aspirants tough

challenge to test their knowledge and at the same time improve skills such as

intelligent guessing, elimination, reasoning, deduction etc – which are much needed

to sail through tough Civil Services Preliminary Examination conducted by UPSC.

These questions are based on this INSTA Revision Plan which is posted on our

website (www.insightsonindia.com). Every year thousands of candidates follow our

revision timetable – which is made for SERIOUS aspirants who would like to intensively

revise everything that’s important before the exam.

Those who would like to take up more tests for even better preparation, can

enroll to Insights IAS Prelims Mock Test Series – 2020

(https://prelims.insightsonindia.com). Every year toppers solve our tests and sail

through UPSC civil services exam. Your support through purchase of our tests will help

us provide FREE content on our website seamlessly.

Wish you all the best!

Team InsightsIAS

Page 3: SIMPLYFYING IAS EXAM PREPARATION...• He also founded the organisation Shri Mathura Vrindavan Hasanand Gochar Bhoomi in Vrindavan. 5. Periyar E. V. Ramasamy was associated with which

INSTA 75 Days REVISION PLAN for Prelims 2020 - InstaTests

www.insightsonindia.com 1 Insights IAS

DAY – 33 (InstaTest-33)

1. Consider the following statements regarding National Policy on Biofuels -2018

1. It envisages an indicative target of 10% blending of ethanol in petrol and 5%

blending of bio-diesel in diesel.

2. It expands the scope of raw material for ethanol production.

3. To ensure food security, the policy does not allow use of food grains for

production of ethanol.

Which of the statements given above is/are correct?

(a) 1 and 2 only

(b) 1 only

(c) 2 and 3 only

(d) 2 only

Solution: D

The National Policy on Biofuels-2018 approved by the Government envisages an indicative

target of 20% blending of ethanol in petrol and 5% blending of bio-diesel in diesel by 2030.

National Policy on biofuels- salient features:

1. Categorization: The Policy categorises biofuels as “Basic Biofuels” viz. First Generation

(1G) bioethanol & biodiesel and “Advanced Biofuels” – Second Generation (2G)

ethanol, Municipal Solid Waste (MSW) to dropin fuels, Third Generation (3G) biofuels,

bio-CNG etc. to enable extension of appropriate financial and fiscal incentives under

each category.

2. Scope of raw materials: The Policy expands the scope of raw material for ethanol

production by allowing use of Sugarcane Juice, Sugar containing materials like Sugar

Beet, Sweet Sorghum, Starch containing materials like Corn, Cassava, Damaged food

grains like wheat, broken rice, Rotten Potatoes, unfit for human consumption for

ethanol production.

3. Protection to farmers: Farmers are at a risk of not getting appropriate price for their

produce during the surplus production phase. Taking this into account, the Policy

allows use of surplus food grains for production of ethanol for blending with petrol

with the approval of National Biofuel Coordination Committee

4. Viability gap funding: With a thrust on Advanced Biofuels, the Policy indicates a

viability gap funding scheme for 2G ethanol Bio refineries of Rs.5000 crore in 6 years

in addition to additional tax incentives, higher purchase price as compared to 1G

biofuels.

Page 4: SIMPLYFYING IAS EXAM PREPARATION...• He also founded the organisation Shri Mathura Vrindavan Hasanand Gochar Bhoomi in Vrindavan. 5. Periyar E. V. Ramasamy was associated with which

INSTA 75 Days REVISION PLAN for Prelims 2020 - InstaTests

www.insightsonindia.com 2 Insights IAS

5. Boost to biodiesel production: The Policy encourages setting up of supply chain

mechanisms for biodiesel production from non-edible oilseeds, Used Cooking Oil,

short gestation crops.

2. Consider the following statements regarding Lala Lajpat Rai

1. He founded the Indian Home Rule League of America in New York City in 1917.

2. He died at Lahore in 1928 after he was attacked by police during a protest rally

against the Simon Commission.

3. Kesari and New India were the important works of him.

Which of the statements given above is/are correct?

(a) 1 and 2 only

(b) 2 and 3 only

(c) 1 and 3 only

(d) 1, 2 and 3

Solution: A

Lala Lajpat Rai

• Lala Lajpat Rai is remembered for his role during the Swadeshi movement and for his

advocacy of education. He became a follower of Dayanand Saraswati, the founder

of the Arya Samaj, and went on to become one of the society’s leaders. He also helped

found the Punjab National Bank.

• In 1885, Rai established the Dayanand Anglo-Vedic School in Lahore and remained a

committed educationist throughout his life.

• Rai, Tilak, and Bipin Chandra Pal (called Lal-Bal-Pal) fervently advocated the use of

Swadeshi goods and mass agitation in the aftermath of the controversial Partition of

Bengal in 1905 by Lord Curzon.

• He founded the Indian Home Rule League of America in New York City in 1917.

• He was elected President of the Indian National Congress during its Special Session

in Kolkata in 1920, which saw the launch of Mahatma Gandhi’s Non-cooperation

Movement.

• The patriot died at Lahore in 1928 after he was attacked by police during a protest

rally against the Simon Commission.

• His important works include: ‘The Arya Samaj’, ‘Young India’, ‘England’s Debt to

India’, ‘Evolution of Japan’, ‘India’s Will to Freedom’, ‘Message of the Bhagwad Gita’,

‘Political Future of India’, ‘Problem of National Education in India’, ‘The Depressed

Glasses’, and the travelogue ‘United States of America’.

Page 5: SIMPLYFYING IAS EXAM PREPARATION...• He also founded the organisation Shri Mathura Vrindavan Hasanand Gochar Bhoomi in Vrindavan. 5. Periyar E. V. Ramasamy was associated with which

INSTA 75 Days REVISION PLAN for Prelims 2020 - InstaTests

www.insightsonindia.com 3 Insights IAS

3. Consider the following statements regarding Anti-Satellite Missile in India

1. It’s essentially a missile that can destroy or jam an enemy country’s satellite in

space.

2. It is a ISRO-developed Ballistic Missile Defence (BMD) Interceptor Missile which

successfully engaged an Indian orbiting target satellite in Low Earth Orbit (LEO)

Which of the statements given above is/are correct?

(a) 1 only

(b) 2 only

(c) Both 1 and 2

(d) Neither 1 nor 2

Solution: A

Anti-Satellite (ASAT) Missile

• It’s essentially a missile that can destroy or jam an enemy country’s satellite in space.

• It is a DRDO-developed Ballistic Missile Defence (BMD) Interceptor Missile which

successfully engaged an Indian orbiting target satellite in Low Earth Orbit (LEO) in a

‘Hit to Kill’ mode.

https://www.thehindu.com/news/national/anti-satellite-missile-capability-showcased-in-r-

day-parade/article30657451.ece

4. Consider the following statements regarding Pt. Madan Mohan Malaviya

1. He had not served as the President of the Indian National Congress.

2. He was the editor of a Hindi magazine, ‘Hindosthan’.

3. He was a social reformer who opposed untouchability and worked for the temple

entry of Dalits.

Which of the statements given above is/are correct?

(a) 1 and 2 only

(b) 2 and 3 only

(c) 1 and 3 only

(d) 1, 2 and 3

Solution: B

Page 6: SIMPLYFYING IAS EXAM PREPARATION...• He also founded the organisation Shri Mathura Vrindavan Hasanand Gochar Bhoomi in Vrindavan. 5. Periyar E. V. Ramasamy was associated with which

INSTA 75 Days REVISION PLAN for Prelims 2020 - InstaTests

www.insightsonindia.com 4 Insights IAS

Pt. Madan Mohan Malaviya

• Pt. Madan Mohan Malaviya was a freedom fighter and social reformer. He had served

as the President of the INC on four occasions. He was awarded the Bharat Ratna

posthumously in 2014.

• He was the editor of a Hindi magazine, ‘Hindosthan’. He became the editor of the

‘Indian Opinion’ in 1889. He also started a Hindi weekly ‘Abhyudaya’, an English daily

‘Leader’, a Hindi newspaper ‘Maryada’.

• Pandit Malaviya was instrumental in founding the Benares Hindu University in 1916.

He also became its Vice-Chancellor till 1939.

• He was opposed to separate electorates to Muslims and the Lucknow Pact.

• He was also against the INC’s participation in the Khilafat Movement.

• He was a participant in the Second Round Table Conference in 1931.

• He started the Ganga Mahasabha to oppose construction of dams in the Ganga.

• He was also a social reformer who opposed untouchability. He worked for the temple

entry of Dalits at the Kalaram Temple at Nashik, Maharashtra.

• He also founded the organisation Shri Mathura Vrindavan Hasanand Gochar Bhoomi

in Vrindavan.

5. Periyar E. V. Ramasamy was associated with which of following movements/Parties?

1. Vaikom Satyagraha

2. Justice Party

3. Self-Respect Movement

4. Dravidar Kazhagam

Select the correct answer using the code given below

(a) 1, 2 and 3 only

(b) 2, 3 and 4 only

(c) 1, 3 and 4 only

(d) 1, 2, 3 and 4

Solution: D

Periyar E. V. Ramasamy

• Born in 1879, Periyar is remembered for the Self Respect Movement to redeem the

identity and self-respect of Tamils. He envisaged a Dravida homeland of Dravida

Nadu, and launched a political party, Dravidar Kazhagam (DK).

• He associated himself with the Justice Party and the Self Respect Movement.

Page 7: SIMPLYFYING IAS EXAM PREPARATION...• He also founded the organisation Shri Mathura Vrindavan Hasanand Gochar Bhoomi in Vrindavan. 5. Periyar E. V. Ramasamy was associated with which

INSTA 75 Days REVISION PLAN for Prelims 2020 - InstaTests

www.insightsonindia.com 5 Insights IAS

• Periyar’s fame spread beyond the Tamil region during the Vaikom Satyagraha of 1924,

a mass movement to demand that lower caste persons be given the right to use a

public path in front of the famous Vaikom temple.

• In the 1940s, Periyar launched Dravidar Kazhagam, which espoused an independent

Dravida Nadu comprising Tamil, Malayalam, Telugu, and Kannada speakers.

• As a social reformer, he focused on social, cultural and gender inequalities, and his

reform agenda questioned matters of faith, gender and tradition. He asked people to

be rational in their life choices. He argued that women needed to be independent, not

mere child-bearers, and insisted that they be allowed an equal share in employment.

6. Operation Vanilla, sometime seen in the news, is

(a) To assist the population of Australia affected by Bush fires

(b) To assist the stranded Indians in the city of Wuhan

(c) To assist the population of Madagascar affected by Cyclone Diane

(d) None of the statements (a), (b) and (c) are correct.

Solution: C

• Operation Vanilla is launched by Indian Navy in the Southern Indian Ocean. The

Operation has been launched to assist the population of Madagascar that were

affected by Cyclone Diane.

https://www.thehindu.com/news/national/indian-navy-launches-operation-vanilla-to-help-

flood-hit-madagascar/article30677388.ece

7. Consider the following statements regarding Indian National Congress

1. The first session of the Indian National Congress was presided over by Womesh

Chandra Bonnerjee.

2. The first session of the Indian National Congress was held in Calcutta in December

1885.

3. Kadambini Ganguly was the first woman president to address the Congress

session.

Which of the statements given above is/are correct?

(a) 1 only

(b) 2 and 3 only

(c) 1 and 3 only

Page 8: SIMPLYFYING IAS EXAM PREPARATION...• He also founded the organisation Shri Mathura Vrindavan Hasanand Gochar Bhoomi in Vrindavan. 5. Periyar E. V. Ramasamy was associated with which

INSTA 75 Days REVISION PLAN for Prelims 2020 - InstaTests

www.insightsonindia.com 6 Insights IAS

(d) 1, 2 and 3

Solution: A

• The final shape to the of all-India organisation idea was given by a retired English civil

servant, A.O. Hume, who mobilised leading intellectuals of the time and, with their

cooperation, organised the first session of the Indian National Congress at Gokuldas

Tejpal Sanskrit College in Bombay in December 1885.

• As a prelude to this, two sessions of the Indian National Conference had been held in

1883 and 1885, which had representatives drawn from all major towns of India.

Surendranath Banerjea and Ananda Mohan Bose were the main architects of the

Indian National Conference.

• The first session of the Indian National Congress was attended by 72 delegates and

presided over by Womesh Chandra Bonnerjee.

• In 1890, Kadambini Ganguly, the first woman graduate of Calcutta University,

addressed the Congress session, which symbolised the commitment of the freedom

struggle to give the women of India their due status in national life.

8. Consider the following statements regarding Chabahar port

1. The port provides access to the resources and markets of landlocked Afghanistan

and Central Asia bypassing Pakistan.

2. Chabahar port will be linked to International North-South Transport Corridor

(INSTC)

Which of the statements given above is/are correct?

(a) 1 only

(b) 2 only

(c) Both 1 and 2

(d) Neither 1 nor 2

Solution: C

• The Chabahar port project will be the first overseas venture for Indian state-owned

ports such as JNPT and the Deendayal Port Trust. It is located in the Sistan-

Baluchistan Province on Iran’s South-eastern coast (outside the Persian Gulf).

• The port provides access to the resources and markets of landlocked Afghanistan and

Central Asia bypassing Pakistan.

Page 9: SIMPLYFYING IAS EXAM PREPARATION...• He also founded the organisation Shri Mathura Vrindavan Hasanand Gochar Bhoomi in Vrindavan. 5. Periyar E. V. Ramasamy was associated with which

INSTA 75 Days REVISION PLAN for Prelims 2020 - InstaTests

www.insightsonindia.com 7 Insights IAS

• Chabahar port will be linked to International North-South Transport Corridor (INSTC),

thus serve as India’s outreach in region to Afghanistan and beyond to Russia, Eastern

Europe.

9. Which of the following resolutions was/were discussed in Surat session, which

eventually led to the split of Congress?

1. Swadeshi

2. Boycott

3. National Education

Select the correct answer using the code given below

(a) 1 and 2 only

(b) 2 and 3 only

(c) 1 and 3 only

(d) 1, 2 and 3

Solution: D

• The Congress split at Surat came in December 1907, around the time when

revolutionary activity had gained momentum.

• The two events were not unconnected. In December 1905, at the Benaras session of

the Indian National Congress presided over by Gokhale, the Moderate-Extremist

differences came to the fore.

Page 10: SIMPLYFYING IAS EXAM PREPARATION...• He also founded the organisation Shri Mathura Vrindavan Hasanand Gochar Bhoomi in Vrindavan. 5. Periyar E. V. Ramasamy was associated with which

INSTA 75 Days REVISION PLAN for Prelims 2020 - InstaTests

www.insightsonindia.com 8 Insights IAS

• The Extremists wanted a strong resolution supporting their programme at the Benaras

session. The Moderates, on the other hand, were not in favour of extending the

movement beyond Bengal and were totally opposed to boycott of councils and similar

associations.

• At the Calcutta session of the Congress in December 1906, the Moderate enthusiasm

had cooled a bit because of the popularity of the Extremists and the revolutionaries

and because of communal riots. Here, the Extremists wanted either Tilak or Lajpat

Rai as the president, while the Moderates proposed the name of Dadabhai Naoroji,

who was widely respected by all the nationalists.

• The Extremists wanted the 1907 session to be held in Nagpur (Central Provinces) with

Tilak or Lajpat Rai as the president along with a reiteration of the swadeshi, boycott

and national education resolutions. The Moderates wanted the session at Surat in

order to exclude Tilak from the presidency, since a leader from the host province

could not be session president (Surat being in Tilak’s home province of Bombay).

• Instead, they wanted Rashbehari Ghosh as the president and sought to drop the

resolutions on swadeshi, boycott and national education. Both sides adopted rigid

positions, leaving no room for compromise.

10. Consider the following statements regarding Anushilan Samiti

1. It was founded by Promotha Mitter.

2. Yugantar was the weekly publication of Anushilan Samiti

Which of the statements given above is/are correct?

(a) 1 only

(b) 2 only

(c) Both 1 and 2

(d) Neither 1 nor 2

Solution: C

• By the 1870s, Calcutta’s student community was honeycombed with secret societies,

but these were not very active. The first revolutionary groups were organised in 1902

in Midnapore (under Jnanendranath Basu) and in Calcutta (the Anushilan Samiti

founded by Promotha Mitter, and including Jatindranath Banerjee, Barindra Kumar

Ghosh and others.)

• But their activities were limited to giving physical and moral training to the members

and remained insignificant till 1907-08.

Page 11: SIMPLYFYING IAS EXAM PREPARATION...• He also founded the organisation Shri Mathura Vrindavan Hasanand Gochar Bhoomi in Vrindavan. 5. Periyar E. V. Ramasamy was associated with which

INSTA 75 Days REVISION PLAN for Prelims 2020 - InstaTests

www.insightsonindia.com 9 Insights IAS

• In April 1906, an inner circle within Anushilan (Barindra Kumar Ghosh,

Bhupendranath Dutta) started the weekly Yugantar and conducted a few abortive

‘actions’.

11. Consider the following statements regarding Komagata Maru Incident

1. Komagata Maru was the name of the Japanese steamship which was carrying the

passengers.

2. It was carrying passengers from India to Vancouver and were turned back by

Canadian authorities.

Which of the statements given above is/are correct?

(a) 1 only

(b) 2 only

(c) Both 1 and 2

(d) Neither 1 nor 2

Solution: A

Komagata Maru Incident

• The importance of this event lies in the fact that it created an explosive situation in

the Punjab. Komagata Maru was the name of a ship which was carrying 370

passengers, mainly Sikh and Punjabi Muslim would-be immigrants, from Singapore to

Vancouver.

• They were turned back by Canadian authorities after two months of privation and

uncertainty. It was generally believed that the Canadian authorities were influenced

by the British government. The ship finally anchored at Calcutta in September 1914.

The inmates refused to board the Punjab bound train. In the ensuing conflict with the

police at Budge Budge near Calcutta, 22 persons died.

• The Komagata Maru incident involved the Japanese steamship Komagata Maru, on

which a group of people from British India attempted to emigrate to Canada in 1914,

but most were denied entry and forced to return to Budge Budge Calcutta (present-

day Kolkata), India.

12. Consider the following statements regarding Home Rule Movement

1. It was the Indian response to the First World War.

2. Indian Home Rule Leagues were organised on the lines of the Russian Home Rule

Leagues.

Page 12: SIMPLYFYING IAS EXAM PREPARATION...• He also founded the organisation Shri Mathura Vrindavan Hasanand Gochar Bhoomi in Vrindavan. 5. Periyar E. V. Ramasamy was associated with which

INSTA 75 Days REVISION PLAN for Prelims 2020 - InstaTests

www.insightsonindia.com 10 Insights IAS

3. Besant got the approval from Congress to setup Home Rule Leagues and many

congress leaders joined the movement.

Which of the statements given above is/are correct?

(a) 1 only

(b) 2 and 3 only

(c) 1 and 3 only

(d) 1, 2 and 3

Solution: A

The Home Rule Movement was the Indian response to the First World War in a less charged

but a more effective way than the response of Indians living abroad which took the form of

the romantic Ghadr adventure.

• Two Indian Home Rule Leagues were organised on the lines of the Irish Home Rule

Leagues and they represented the emergence of a new trend of aggressive politics.

• Annie Besant and Tilak were the pioneers of this new trend. Although Besant failed

to get the Congress to approve her scheme of Home Rule Leagues, the Congress did

commit itself to a programme of educative propaganda and to a revival of local-level

Congress committees.

• Not willing to wait for too long, Besant laid the condition that if the Congress did not

implement its commitments, she would be free to set up her own league—which she

finally had to, as there was no response from the Congress.

• Tilak and Besant set up their separate leagues to avoid any friction.

• Tilak set up his Home Rule League in April 1916 and it was restricted to Maharashtra

(excluding Bombay city), Karnataka, Central Provinces and Berar. It had six branches

and the demands included swarajya, formation of linguistic states and education in

the vernacular.

13. Which of the following was/were the reforms under Government of India Act, 1919

1. Women were given the right to vote.

2. A bicameral arrangement was introduced.

3. The legislative councils could initiate legislation

Select the correct answer using the code given below

(a) 1 only

(b) 2 and 3 only

Page 13: SIMPLYFYING IAS EXAM PREPARATION...• He also founded the organisation Shri Mathura Vrindavan Hasanand Gochar Bhoomi in Vrindavan. 5. Periyar E. V. Ramasamy was associated with which

INSTA 75 Days REVISION PLAN for Prelims 2020 - InstaTests

www.insightsonindia.com 11 Insights IAS

(c) 1 and 2 only

(d) 1, 2 and 3

Solution: D

In line with the government policy contained in Montagu’s statement of August 1917, the

government announced further constitutional reforms in July 1918, known as Montagu-

Chelmsford or Montford Reforms. Based on these, the Government of India Act, 1919 was

enacted.

Legislature

1) Provincial legislative councils were further expanded and 70 per cent of the members

were to be elected.

2) The system of communal and class electorates was further consolidated.

3) Women were also given the right to vote.

4) The legislative councils could initiate legislation but the governor’s assent was

required. The governor could veto bills and issue ordinances.

5) The legislative councils could reject the budget but the governor could restore it, if

necessary.

6) The legislators enjoyed freedom of speech.

Legislature

1) A bicameral arrangement was introduced.

2) The Council of State had a tenure of 5 years and had only male members, while the

Central Legislative Assembly had a tenure of 3 years.

3) The legislators could ask questions and supplementaries, pass adjournment motions

and vote a part of the budget, but 75 per cent of the budget was still not votable.

14. Which of the following is/are considered as Contempt of Court?

1. Disobeying court orders,

2. Interfering with judicial proceedings,

3. Obstructing the administration of justice

4. Scandalizing or lowering the authority of the court

Select the correct answer using the code given below

(a) 1, 2 and 3 only

(b) 2, 3 and 4 only

(c) 1, 2 and 4 only

(d) 1, 2, 3 and 4

Page 14: SIMPLYFYING IAS EXAM PREPARATION...• He also founded the organisation Shri Mathura Vrindavan Hasanand Gochar Bhoomi in Vrindavan. 5. Periyar E. V. Ramasamy was associated with which

INSTA 75 Days REVISION PLAN for Prelims 2020 - InstaTests

www.insightsonindia.com 12 Insights IAS

Solution: D

• The Supreme Court has held that courts are empowered to order parties in a

contempt case to surrender their passport in order to ensure their presence in the

proceedings.

Background:

• The court was hearing a contempt case in connection with a civil suit on partition of a

disputed property in Delhi.

What is contempt under the Indian law?

In India, the Contempt of Courts Act, 1971, divides contempt into civil contempt and criminal

contempt.

1. ‘Civil contempt’ is a ‘wilful disobedience to any judgment, decree, direction, order,

writ or other processes of a Court or wilful breach of an undertaking given to the

court’.

2. ‘Criminal contempt’ is ‘the publication (whether by words, spoken or written, or by

signs, or by visible representation, or otherwise) of any matter or the doing of any

other act whatsoever which:

1) Scandalises or tends to scandalise, or lowers or tends to lower the authority

of, any court.

2) Prejudices, or interferes or tends to interfere with the due course of any

judicial proceeding.

3) Interferes or tends to interfere with, or obstructs or tends to obstruct, the

administration of justice in any other manner.’

Contempt of Courts (Amendment) Act, 2006:

• The statute of 1971 has been amended by the Contempt of Courts (Amendment) Act,

2006 to include the defence of truth under Section 13 of the original legislation.

• Section 13 that already served to restrict the powers of the court in that they were

not to hold anyone in contempt unless it would substantially interfere with the due

process of justice, the amendment further states that the court must permit

‘justification by truth as a valid defence if it is satisfied that it is in public interest and

the request for invoking the said defence is bona fide.’

Constitutional Background:

• Article 129: Grants Supreme Court the power to punish for contempt of itself.

• Article 142(2): Enables the Supreme Court to investigate and punish any person for

its contempt.

• Article 215: Grants every High Court the power to punish for contempt of itself.

Page 15: SIMPLYFYING IAS EXAM PREPARATION...• He also founded the organisation Shri Mathura Vrindavan Hasanand Gochar Bhoomi in Vrindavan. 5. Periyar E. V. Ramasamy was associated with which

INSTA 75 Days REVISION PLAN for Prelims 2020 - InstaTests

www.insightsonindia.com 13 Insights IAS

15. Consider the following statements regarding Convalescent Plasma Therapy (CPT)

1. It seeks to make use of the antibodies developed from genetic engineering

technique.

2. Therapy is useful in treating the patients infected with corona and other type of

viruses.

Which of the statements given above is/are correct?

(a) 1 only

(b) 2 only

(c) Both 1 and 2

(d) Neither 1 nor 2

Solution: B

• It seeks to make use of the antibodies developed in the recovered patient against the

coronavirus.

• The whole blood or plasma from such people is taken, and the plasma is then injected

in critically ill patients so that the antibodies are transferred and boost their fight

against the virus.

• A study in The Lancet Infectious Diseases said a COVID-19 patient usually develops

primary immunity against the virus in 10-14 days.

• Therefore, if the plasma is injected at an early stage, it can possibly help fight the virus

and prevent severe illness.

https://indianexpress.com/article/explained/recovered-covid-19-patients-plasma-what-is-

this-experimental-therapy-6337764/

16. Consider the following statements regarding Champaran Satyagraha

1. Gandhi was requested by Rajkumar Shukla to look into the problems of the

farmers in Champaran.

2. It was a no-tax peasant struggle and demand for plague bonus.

3. The government appointed a committee to go into the matter and nominated

Gandhi as a member.

Which of the statements given above is/are correct?

(a) 1 only

(b) 2 and 3 only

(c) 1 and 3 only

Page 16: SIMPLYFYING IAS EXAM PREPARATION...• He also founded the organisation Shri Mathura Vrindavan Hasanand Gochar Bhoomi in Vrindavan. 5. Periyar E. V. Ramasamy was associated with which

INSTA 75 Days REVISION PLAN for Prelims 2020 - InstaTests

www.insightsonindia.com 14 Insights IAS

(d) 1, 2 and 3

Solution: C

Champaran Satyagraha (1917)—First Civil Disobedience

• Gandhi was requested by Rajkumar Shukla, a local man, to look into the problems of

the farmers in context of indigo planters of Champaran in Bihar. The European

planters had been forcing the peasants to grow indigo on 3/20 part of the total land

(called tinkathia system).

• When towards the end of the nineteenth century German synthetic dyes replaced

indigo, the European planters demanded high rents and illegal dues from the

peasants in order to maximize their profits before the peasants could shift to other

crops. Besides, the peasants were forced to sell the produce at prices fixed by the

Europeans.

• When Gandhi, joined now by Rajendra Prasad, Mazharul- Haq, Mahadeo Desai,

Narhari Parekh, and J.B. Kripalani, reached Champaran to probe into the matter, the

authorities ordered him to leave the area at once.

• Gandhi defied the order and preferred to face the punishment. This passive resistance

or civil disobedience of an unjust order was a novel method at that time. Finally, the

authorities retreated and permitted Gandhi to make an enquiry.

• Now, the government appointed a committee to go into the matter and nominated

Gandhi as a member. Gandhi was able to convince the authorities that the tinkathia

system should be abolished and that the peasants should be compensated for the

illegal dues extracted from them. As a compromise with the planters, he agreed that

only 25 per cent of the money taken should be compensated.

17. Gender Social Norms Index has been released by

(a) World Economic Forum (WEF)

(b) Organisation of Economic Cooperation and Development (OECD)

(c) UN Development Programme (UNDP)

(d) United Nations Educational, Scientific and Cultural Organization (UNESCO)

Solution: C

• The first Gender Social Norms Index was recently released by the UN Development

Programme (UNDP).

Page 17: SIMPLYFYING IAS EXAM PREPARATION...• He also founded the organisation Shri Mathura Vrindavan Hasanand Gochar Bhoomi in Vrindavan. 5. Periyar E. V. Ramasamy was associated with which

INSTA 75 Days REVISION PLAN for Prelims 2020 - InstaTests

www.insightsonindia.com 15 Insights IAS

About the index:

• This index measures how social beliefs obstruct gender equality in areas like politics,

work, and education, and contains data from 75 countries, covering over 80 percent

of the world’s population.

• The index found new clues to the invisible barriers women face in achieving equality

– potentially forging a path forward to breaking through the so-called “glass ceiling”.

Key findings:

• Despite decades of progress closing the equality gap between men and women, close

to 90 percent of men and women hold some sort of bias against women.

• Almost half of those polled feel that men are superior political leaders.

• More than 40 per cent believe they make better business executives and are more

entitled to jobs when the economy is lagging.

• Moreover, 28 per cent think it is justified for a man to beat his wife.

• The analysis also highlighted a bias shift in some 30 countries, revealing that while

some show improvements, attitudes in others appear to have worsened in recent

years – signaling that progress cannot be taken for granted.

Page 18: SIMPLYFYING IAS EXAM PREPARATION...• He also founded the organisation Shri Mathura Vrindavan Hasanand Gochar Bhoomi in Vrindavan. 5. Periyar E. V. Ramasamy was associated with which

INSTA 75 Days REVISION PLAN for Prelims 2020 - InstaTests

www.insightsonindia.com 16 Insights IAS

18. Consider the following statements regarding The Anarchical and Revolutionary Crimes

Act

1. It was based on the recommendations made by the Rowlatt Commission.

2. The act allowed political activists to be tried without juries or even imprisoned

without trial.

3. The law of habeas corpus was sought to be suspended.

Which of the statements given above is/are correct?

(a) 1 and 2 only

(b) 2 and 3 only

(c) 1 and 3 only

(d) 1, 2 and 3

Solution: D

• The Anarchical and Revolutionary Crimes Act, popularly known as the Rowlatt Act,

was based on the recommendations made in the previous year to the Imperial

Legislative Council by the Rowlatt Commission, headed by the British judge, Sir Sidney

Rowlatt, to investigate the ‘seditious conspiracy’ of the Indian people. (The committee

had recommended that activists should be deported or imprisoned without trial for

two years, and that even possession of seditious newspapers would be adequate

evidence of guilt.)

• All the elected Indian members of the Imperial Legislative Council voted against the

bill but they were in a minority and easily overruled by the official nominees. All the

elected Indian members—who included Mohammed Ali Jinnah, Madan Mohan

Malaviya and Mazhar Ul Haq – resigned in protest.

• The act allowed political activists to be tried without juries or even imprisoned without

trial. It allowed arrest of Indians without warrant on the mere suspicion of ‘treason’.

Such suspects could be tried in secrecy without recourse to legal help. A special cell

consisting of three high court judges was to try such suspects and there was no court

of appeal above that panel. This panel could even accept evidence not acceptable

under the Indian Evidences Act.

• The law of habeas corpus, the basis of civil liberty, was sought to be suspended.

19. Which of the following personalities was/ were Swarajists during India’s freedom

struggle?

1. Rajendra Prasad

2. C.R. Das

3. Motilal Nehru

Page 19: SIMPLYFYING IAS EXAM PREPARATION...• He also founded the organisation Shri Mathura Vrindavan Hasanand Gochar Bhoomi in Vrindavan. 5. Periyar E. V. Ramasamy was associated with which

INSTA 75 Days REVISION PLAN for Prelims 2020 - InstaTests

www.insightsonindia.com 17 Insights IAS

4. Ajmal Khan

Select the correct answer using the code given below

(a) 1, 2 and 3 only

(b) 2, 3 and 4 only

(c) 1, 3 and 4 only

(d) 1, 2, 3 and 4

Solution: B

• One section led by C.R. Das, Motilal Nehru and Ajmal Khan wanted an end to the

boycott of legislative councils so that the nationalists could enter them to expose the

basic weaknesses of these assemblies and use these councils as an arena of political

struggle to arouse popular enthusiasm.

• They wanted, in other words, to ‘end or mend’ these councils, i.e., if the government

did not respond to the nationalists’ demands, then they would obstruct the working

of these councils.

• Those advocating entry into legislative councils came to be known as the ‘Swarajists’,

while the other school of thought led by C. Rajagopalachari, Vallabhbhai Patel,

Rajendra Prasad and M.A. Ansari came to be known as the ‘Nochangers’.

• The ‘No-changers’ opposed council entry, advocated concentration on constructive

work, and continuation of boycott and non-cooperation, and quiet preparation for

resumption of the suspended civil disobedience programme.

20. Consider the following statements regarding North East Venture Fund (NEVF)

1. It is set up by North Eastern Council in association with Ministry of Development

of North Eastern Region (M-DoNER).

2. It is the first dedicated venture capital fund for the North Eastern Region.

3. It aims to provide funding for the tribal women entrepreneurs of North Eastern

region.

Which of the statements given above is/are correct?

(a) 1 and 2 only

(b) 1 and 3 only

(c) 2 only

(d) 1, 2 and 3

Page 20: SIMPLYFYING IAS EXAM PREPARATION...• He also founded the organisation Shri Mathura Vrindavan Hasanand Gochar Bhoomi in Vrindavan. 5. Periyar E. V. Ramasamy was associated with which

INSTA 75 Days REVISION PLAN for Prelims 2020 - InstaTests

www.insightsonindia.com 18 Insights IAS

Solution: C

About North East Venture Fund (NEVF):

• Launched in September 2017.

• Set up by North Eastern Development Finance Corporation Limited (NEDFi) in

association with Ministry of Development of North Eastern Region (M-DoNER).

• It is a close ended fund with capital commitment of Rs 100 crore.

• It is the first dedicated venture capital fund for the North Eastern Region.

• Objective: to contribute to the entrepreneurship development of the NER and

achieve attractive risk-adjusted returns through long term capital appreciation by way

of investments in privately negotiated equity/ equity related investments.

• The investment under this schemer ranges from Rs. 25 lakh to Rs.10 crore per

venture, which is long term in nature with investment horizon of 4-5 years.

21. Consider the following statements regarding Mudumalai Tiger Reserve

1. It shares its boundaries with the states of Karnataka and Kerala.

2. It is a part of Nilgiri Biosphere Reserve

Which of the statements given above is/are correct?

(a) 1 only

(b) 2 only

(c) Both 1 and 2

(d) Neither 1 nor 2

Solution: C

• The Mudumalai National Park and Wildlife Sanctuary also a declared tiger reserve,

lies on the northwestern side of the Nilgiri Hills in Nilgiri District, in Tamil Nadu, India.

It shares its boundaries with the states of Karnataka and Kerala.

• It is a part of Nilgiri Biosphere Reserve along with Wayanad Wildlife Sanctuary

(Kerala) in the West, Bandipur National Park (Karnataka) in the North, Mukurthi

National Park and Silent Valley in the South. The sanctuary is divided into five ranges

– Masinagudi, Thepakadu, Mudumalai, Kargudi and Nellakota.

Page 21: SIMPLYFYING IAS EXAM PREPARATION...• He also founded the organisation Shri Mathura Vrindavan Hasanand Gochar Bhoomi in Vrindavan. 5. Periyar E. V. Ramasamy was associated with which

INSTA 75 Days REVISION PLAN for Prelims 2020 - InstaTests

www.insightsonindia.com 19 Insights IAS

22. Which of the following is/are the recommendations of Nehru Report?

1. Dominion status on lines of self-governing dominions desired by Indians

2. Rejection of separate electorates

3. Linguistic provinces

4. Complete dissociation of State from religion

Select the correct answer using the code given below

(a) 1, 2 and 3 only

(b) 2, 3 and 4 only

(c) 1, 3 and 4 only

(d) 1, 2, 3 and 4

Solution: D

The Nehru Report confined itself to British India, as it envisaged the future link-up of British

India with the princely states on a federal basis. For the dominion it recommended:

Page 22: SIMPLYFYING IAS EXAM PREPARATION...• He also founded the organisation Shri Mathura Vrindavan Hasanand Gochar Bhoomi in Vrindavan. 5. Periyar E. V. Ramasamy was associated with which

INSTA 75 Days REVISION PLAN for Prelims 2020 - InstaTests

www.insightsonindia.com 20 Insights IAS

• Dominion status on lines of self-governing dominions as the form of government

desired by Indians (much to the chagrin of younger, militant section—Nehru being

prominent among them).

• Rejection of separate electorates which had been the basis of constitutional reforms

so far; instead, a demand for joint electorates with reservation of seats for Muslims.

• Linguistic provinces.

• Nineteen fundamental rights including equal rights for women, right to form unions,

and universal adult suffrage.

• Responsible government at the Centre and in provinces

• Full protection to cultural and religious interests of Muslims.

• Complete dissociation of State from religion.

23. Consider the following statements regarding individual satyagraha

1. The demand of the satyagraha would be the freedom of speech against the war

through an anti-war declaration.

2. It was associated with Delhi Chalo Movement.

Which of the statements given above is/are correct?

(a) 1 only

(b) 2 only

(c) Both 1 and 2

(d) Neither 1 nor 2

Solution: C

• The government had taken the adamant position that no constitutional advance could

be made till the Congress came to an agreement with the Muslim leaders. It issued

ordinance after ordinance taking away the freedom of speech and that of the press

and the right to organise associations.

• Towards the end of 1940, the Congress once again asked Gandhi to take command.

Gandhi now began taking steps which would lead to a mass struggle within his broad

strategic perspective. He decided to initiate a limited satyagraha on an individual basis

by a few selected individuals in every locality.

The aims of launching individual satyagraha were—

1. to show that nationalist patience was not due to weakness;

2. to express people’s feeling that they were not interested in the war and that they

made no distinction between Nazism and the double autocracy that ruled India; and

Page 23: SIMPLYFYING IAS EXAM PREPARATION...• He also founded the organisation Shri Mathura Vrindavan Hasanand Gochar Bhoomi in Vrindavan. 5. Periyar E. V. Ramasamy was associated with which

INSTA 75 Days REVISION PLAN for Prelims 2020 - InstaTests

www.insightsonindia.com 21 Insights IAS

3. to give another opportunity to the government to accept Congress’ demands

peacefully.

The demand of the satyagraha would be the freedom of speech against the war through an

anti-war declaration. If the government did not arrest the satyagrahi, he or she would not

only repeat it but move into villages and start a march towards Delhi, thus precipitating a

movement which came to be known as the ‘Delhi Chalo Movement’.

Vinoba Bhave was the first to offer the satyagraha and Nehru, the second. By May 1941,

25,000 people had been convicted for individual civil disobedience.

24. Consider the following statements regarding Central Pollution Control Board

1. It was established under Air (Prevention and Control of Pollution) Act, 1981.

2. It serves as a field formation and also provides technical services to the Ministry

of Environment and Forests under the provisions of the Environment (Protection)

Act, 1986.

Which of the statements given above is/are correct?

(a) 1 only

(b) 2 only

(c) Both 1 and 2

(d) Neither 1 nor 2

Solution: B

The Central Pollution Control Board (CPCB) of India is a statutory organisation under the

Ministry of Environment, Forest and Climate Change (MoEF&CC). It was established in 1974

under the Water (Prevention and Control of pollution) Act, 1974.

• CPCB is entrusted with the powers and functions under the Air (Prevention and

Control of Pollution) Act, 1981. It serves as a field formation and also provides

technical services to the Ministry of Environment and Forests under the provisions of

the Environment (Protection) Act, 1986. It Co-ordinates the activities of the State

Pollution Control Boards by providing technical assistance and guidance and also

resolves disputes among them.

25. Consider the following statements regarding Desai-Liaqat Pact

1. An equal number of persons nominated by the Congress and the League in the

central legislature for the formation of an interim government

2. Nearly 20% of the seats were reserved for minorities.

Page 24: SIMPLYFYING IAS EXAM PREPARATION...• He also founded the organisation Shri Mathura Vrindavan Hasanand Gochar Bhoomi in Vrindavan. 5. Periyar E. V. Ramasamy was associated with which

INSTA 75 Days REVISION PLAN for Prelims 2020 - InstaTests

www.insightsonindia.com 22 Insights IAS

Which of the statements given above is/are correct?

(a) 1 only

(b) 2 only

(c) Both 1 and 2

(d) Neither 1 nor 2

Solution: C

Desai-Liaqat Pact

Desai-Liaqat Pact was concluded between Bhulabhai Desai of the Congress and Liaqat Ali

Khan of the Muslim League. It was to find out the way out of the 1942-45 political impasses.

Efforts continued to end the deadlock. Bhulabhai Desai, leader of the Congress Party in the

Central Legislative Assembly, met Liaqat Ali Khan, deputy leader of the Muslim League in that

Assembly, and both of them came up with the draft proposal for the formation of an interim

government at the centre, consisting of—

• an equal number of persons nominated by the Congress and the League in the

central legislature.

• 20% reserved seats for minorities.

No settlement could be reached between the Congress and the League on these lines, but the

fact that a sort of parity between the Congress and the League was decided upon had far-

reaching consequences.

DAY – 34 (InstaTest-34)

26. Consider the following statements regarding Sanyasi Revolt

1. The famine of 1770 and the harsh economic order of the British compelled a group

of sanyasis to fight the British.

2. The Muslims didn’t participate in this revolt because of its religious character.

3. Anandamath by Bankim Chandra Chattopadhyay, is based on the Sanyasi Revolt

Which of the statements given above is/are correct?

(a) 1 and 2 only

(b) 2 and 3 only

(c) 1 and 3 only

(d) 1, 2 and 3

Page 25: SIMPLYFYING IAS EXAM PREPARATION...• He also founded the organisation Shri Mathura Vrindavan Hasanand Gochar Bhoomi in Vrindavan. 5. Periyar E. V. Ramasamy was associated with which

INSTA 75 Days REVISION PLAN for Prelims 2020 - InstaTests

www.insightsonindia.com 23 Insights IAS

Solution: C

Sanyasi Revolt (1763-1800)

• The disastrous famine of 1770 and the harsh economic order of the British compelled

a group of sanyasis in Eastern India to fight the British yoke.

• Originally peasants, even some evicted from land, these sanyasis were joined by a

large number of dispossessed small zamindars, disbanded soldiers and rural poor.

They raided Company factories and the treasuries, and fought the Company’s forces.

It was only after a prolonged action that Warren Hastings could subdue the sanyasis.

• Equal participation of Hindus and Muslims characterised the uprisings, sometimes

referred to as the Fakir Rebellion. Majnum Shah (or Majnu Shah), Chirag Ali, Musa

Shah, Bhawani Pathak and Debi Chaudhurani were important leaders. Debi

Chaudhurani’s participation recognizes the women’s role in early resistances against

the British.

• Anandamath, a semi-historical novel by Bankim Chandra Chattopadhyay, is based on

the Sanyasi Revolt. Bankim Chandra also wrote a novel, Devi Chaudhurani, as he saw

the importance of women too taking up the struggle against an alien rule that posed

a threat to traditional Indian values.

27. Consider the following statements regarding Ahom Revolt

1. It was a revolt against the British who attempted to incorporate the Ahom’s’

territories in the Company’s dominion

2. The rebellion was organized under the leadership of Gomdhar Konwar, an Ahom

prince.

Which of the statements given above is/are correct?

(a) 1 only

(b) 2 only

(c) Both 1 and 2

(d) Neither 1 nor 2

Solution: C

Ahom Revolt (1828)

• The British had pledged to withdraw from Assam after the First Burma War (1824-26).

But, after the war, instead of withdrawing, the British attempted to incorporate the

Ahoms’ territories in the Company’s dominion.

Page 26: SIMPLYFYING IAS EXAM PREPARATION...• He also founded the organisation Shri Mathura Vrindavan Hasanand Gochar Bhoomi in Vrindavan. 5. Periyar E. V. Ramasamy was associated with which

INSTA 75 Days REVISION PLAN for Prelims 2020 - InstaTests

www.insightsonindia.com 24 Insights IAS

• This sparked off a rebellion in 1828 under the leadership of Gomdhar Konwar, an

Ahom prince, along with compatriots, such as Dhanjoy Bongohain, and Jairam

Khargharia Phukan. Assembling near Jorhat, the rebels formally made Gomdhar

Konwar the king.

• Finally, the Company decided to follow a conciliatory policy and handed over Upper

Assam to Maharaja Purandar Singh Narendra and part of the kingdom was restored

to the Assamese king.

28. Consider the following statements regarding Kuka Movement

1. The Kuka Movement was founded by Baba Ram Singh.

2. The Kukas wanted to remove the British and restore Sikh rule over Punjab.

3. The concepts of Swadeshi and non-cooperation were propagated by the Kukas.

Which of the statements given above is/are correct?

(a) 1 and 2 only

(b) 2 and 3 only

(c) 1 and 3 only

(d) 1, 2 and 3

Solution: B

Kuka Movement

• The Kuka Movement was founded in 1840 by Bhagat Jawahar Mal (also called Sian

Saheb) in western Punjab. A major leader of the movement after him was Baba Ram

Singh.

• (He founded the Namdhari Sikh sect.) After the British took Punjab, the movement

got transformed from a religious purification campaign to a political campaign. Its

basic tenets were abolition of caste and similar discriminations among Sikhs,

discouraging the consumption of meat and alcohol and drugs, permission for

intermarriages, widow remarriage, and encouraging women to step out of seclusion.

• On the political side, the Kukas wanted to remove the British and restore Sikh rule

over Punjab; they advocated wearing hand-woven clothes and boycott of English laws

and education and products. So, the concepts of Swadeshi and non-cooperation were

propagated by the Kukas, much before they became part of the Indian national

movement in the early twentieth century.

• As the movement gained in popularity, the British took several steps to crush it in the

period between 1863 and 1872.

Page 27: SIMPLYFYING IAS EXAM PREPARATION...• He also founded the organisation Shri Mathura Vrindavan Hasanand Gochar Bhoomi in Vrindavan. 5. Periyar E. V. Ramasamy was associated with which

INSTA 75 Days REVISION PLAN for Prelims 2020 - InstaTests

www.insightsonindia.com 25 Insights IAS

29. Consider the following statements regarding Faraizi Movement

1. The Faraizis were the followers of a Muslim sect founded by Haji Shariat-Allah

2. It aimed at the eradication of social innovations or un-Islamic practices among the

Muslims

3. The movement was merely a religious movement without political overtones.

Which of the statements given above is/are correct?

(a) 1 and 2 only

(b) 2 and 3 only

(c) 1 and 3 only

(d) 1, 2 and 3

Solution: A

Faraizi Movement

• The Faraizis were the followers of a Muslim sect founded by Haji Shariat-Allah of

Faridpur in Eastern Bengal.

• The movement, also called the Fara’idi Movement because of its emphasis on the

Islamic pillars of faith, was founded by Haji Shariatullah in 1818.

• Its scene of action was East Bengal, and it aimed at the eradication of social

innovations or un-Islamic practices current among the Muslims of the region and draw

their attention to their duties as Muslims.

• Under the leadership of Haji’s son, Dudu Mian, the movement became revolutionary

from 1840 onwards. He gave the movement an organisational system from the village

to the provincial level with a khalifa or authorised deputy at every level.

• The Fara’idis organised a paramilitary force armed with clubs to fight the zamindars

who were mostly Hindu, though there were some Muslim landlords too, besides the

indigo planters. Dudu Mian asked his followers not to pay rent. The organisation even

established its own Law courts.

• Dudu Mian was arrested several times, and his arrest in 1847 finally weakened the

movement. The movement survived merely as a religious movement without political

overtones after the death of Dudu Mian in 1862.

30. Consider the following statements regarding Western Ghats

1. They are older than the Himalayas

2. It is one of the eight “hottest hot-spots” of biological diversity in the world

3. They are spanning over five states.

Page 28: SIMPLYFYING IAS EXAM PREPARATION...• He also founded the organisation Shri Mathura Vrindavan Hasanand Gochar Bhoomi in Vrindavan. 5. Periyar E. V. Ramasamy was associated with which

INSTA 75 Days REVISION PLAN for Prelims 2020 - InstaTests

www.insightsonindia.com 26 Insights IAS

4. Kasturirangan Committee has proposed a Western Ghats Ecology Authority to

regulate these activities in this area.

Which of the statements given above is/are correct?

(a) 1, 2 and 3 only

(b) 1 and 2 only

(c) 2 and 4 only

(d) None of the above

Solution: A

• A public interest litigation petition has been filed in the Madras High Court seeking a

direction to the Centre and State government to constitute a permanent body for

taking serious steps to safeguard the flora, fauna and other natural resources in the

Eastern and Western Ghat areas in Tamil Nadu.

• The petition is on the basis of the recommendations made by the Madhav Gadgil and

Kasturi Rangan committees.

Importance of Western Ghats:

• The Western Ghats is an extensive region spanning over six States. It is the home of

many endangered plants and animals. It is a UNESCO World Heritage site.

• It is one of the eight “hottest hot-spots” of biological diversity in the world.

• According to UNESCO, the Western Ghats are older than the Himalayas. They

influence Indian monsoon weather patterns by intercepting the rain-laden monsoon

winds that sweep in from the south-west during late summer.

Gadgil Committee:

• It defined the boundaries of the Western Ghats for the purposes of ecological

management.

• It proposed that this entire area be designated as ecologically sensitive area (ESA).

• Within this area, smaller regions were to be identified as ecologically sensitive zones

(ESZ) I, II or III based on their existing condition and nature of threat.

• It proposed to divide the area into about 2,200 grids, of which 75 per cent would fall

under ESZ I or II or under already existing protected areas such as wildlife sanctuaries

or natural parks.

• The committee proposed a Western Ghats Ecology Authority to regulate these

activities in the area.

Page 29: SIMPLYFYING IAS EXAM PREPARATION...• He also founded the organisation Shri Mathura Vrindavan Hasanand Gochar Bhoomi in Vrindavan. 5. Periyar E. V. Ramasamy was associated with which

INSTA 75 Days REVISION PLAN for Prelims 2020 - InstaTests

www.insightsonindia.com 27 Insights IAS

Page 30: SIMPLYFYING IAS EXAM PREPARATION...• He also founded the organisation Shri Mathura Vrindavan Hasanand Gochar Bhoomi in Vrindavan. 5. Periyar E. V. Ramasamy was associated with which

INSTA 75 Days REVISION PLAN for Prelims 2020 - InstaTests

www.insightsonindia.com 28 Insights IAS

Extra Reading:

Kasturirangan Committee:

• None of the six concerned states agreed with the recommendations of the Gadgil

Committee, which submitted its report in August 2011.

• In August 2012, then Environment Minister constituted a High-Level Working Group

on Western Ghats under Kasturirangan to “examine” the Gadgil Committee report in

a “holistic and multidisciplinary fashion in the light of responses received” from states,

central ministries and others.

• The Kasturirangan report seeks to bring just 37% of the Western Ghats under the

Ecologically Sensitive Area (ESA) zones — down from the 64% suggested by the Gadgil

report.

Recommendations of Kasturirangan Committee:

• A ban on mining, quarrying and sand mining.

• No new thermal power projects, but hydro power projects allowed with restrictions.

• A ban on new polluting industries.

• Building and construction projects up to 20,000 sq m was to be allowed but townships

were to be banned.

• Forest diversion could be allowed with extra safeguards.

31. The ‘United for Biodiversity’ coalition has been launched by

(a) Global Environment Facility

(b) Conservation International

(c) European Commission

(d) World Wide Fund for Nature

Solution: C

• The European Commission (EC) has launched the ‘United for Biodiversity’ coalition.

• It was launched on World Wildlife Day 2020- 3rd March.

What is it?

• The coalition is made up of zoos, aquariums, botanical gardens, national parks, and

natural history and science museums from around the world.

• The coalition offers the opportunity for all such institutions to “join forces and boost

public awareness about the nature crisis, ahead of the crucial COP-15 of the

Convention on Biological Diversity in Kunming, China in October 2020.

Page 31: SIMPLYFYING IAS EXAM PREPARATION...• He also founded the organisation Shri Mathura Vrindavan Hasanand Gochar Bhoomi in Vrindavan. 5. Periyar E. V. Ramasamy was associated with which

INSTA 75 Days REVISION PLAN for Prelims 2020 - InstaTests

www.insightsonindia.com 29 Insights IAS

A common pledge adopted:

• The coalition adopted a common pledge, citing the Intergovernmental Platform on

Biodiversity and Ecosystem Services (IPBES) Global Assessment finding that one

million species were already at risk of extinction, and appeals to visitors to each of

their institutions to “raise their voice for nature.”

32. Consider the following statements regarding Prime Minister’s National Relief Fund

(PMNRF)

1. It has been constituted by the Parliament

2. It is recognized as a Trust under the Indian Trusts Act, 1882

3. The contributions under the PMNRF can be qualified as corporate social

responsibility

Which of the statements given above is/are correct?

(a) 3 only

(b) 2 and 3 only

(c) 1 and 3 only

(d) 1, 2 and 3

Solution: A

Prime Minister’s National Relief Fund (PMNRF):

• In pursuance of an appeal by the then Prime Minister, Pt. Jawaharlal Nehru in

January, 1948, the Prime Minister’s National Relief Fund (PMNRF) was established

with public contributions to assist displaced persons from Pakistan.

• The resources of the PMNRF are now utilized primarily to render immediate relief to

families of those killed in natural calamities like floods, cyclones and earthquakes, etc.

and to the victims of the major accidents and riots.

• Assistance from PMNRF is also rendered, to partially defray the expenses for medical

treatment like heart surgeries, kidney

Key features:

• Disbursements are made with the approval of the Prime Minister.

• PMNRF has not been constituted by the Parliament.

• The fund is recognized as a Trust under the Income Tax Act and the same is managed

by Prime Minister or multiple delegates for national causes.

• PMNRF is exempt under Income Tax Act.

Page 32: SIMPLYFYING IAS EXAM PREPARATION...• He also founded the organisation Shri Mathura Vrindavan Hasanand Gochar Bhoomi in Vrindavan. 5. Periyar E. V. Ramasamy was associated with which

INSTA 75 Days REVISION PLAN for Prelims 2020 - InstaTests

www.insightsonindia.com 30 Insights IAS

• Prime Minister is the Chairman of PMNRF and is assisted by Officers/ Staff on honorary

basis.

• These contributions also qualify as CSR (corporate social responsibility) spend for

companies, making it more attractive in terms of tax exemptions.

33. Consider the following statements regarding The Santhal Rebellion

1. It was a rebellion against the zamindars later turned into an anti-British

movement.

2. It was organized under the leadership of Buddho Bhagat.

Which of the statements given above is/are correct?

(a) 1 only

(b) 2 only

(c) Both 1 and 2

(d) Neither 1 nor 2

Solution: A

The Santhal Rebellion (1855-56)

• Continued oppression of the Santhals, an agricultural people, who had fled to settle

in the plains of the Rajmahal hills (Bihar) led to the Santhal rebellion against the

zamindars.

• The money-lenders who had the support of the police among others had joined the

zamindars to subject the peasants to oppressive exactions and dispossession of lands.

• The rebellion turned into an anti-British movement. Under Sidhu and Kanhu, two

brothers, the Santhals proclaimed an end to Company rule, and declared the area

between Bhagalpur and Rajmahal as autonomous. The rebellion was suppressed by

1856.

• The Kols rebellion in 1831 was under the leadership of Buddho Bhagat.

34. Consider the following statements regarding Raja Rammohan Roy

1. He had set up the Atmiya Sabha and Brahmo Sabha.

2. The long-term agenda of Samaj was to purify Hinduism, to preach monotheism

and to establish a new religion.

3. Raja Radhakant Deb organised the Dharma Sabha to propagate the ideals of

Brahmo Samaj.

Which of the statements given above is/are correct?

Page 33: SIMPLYFYING IAS EXAM PREPARATION...• He also founded the organisation Shri Mathura Vrindavan Hasanand Gochar Bhoomi in Vrindavan. 5. Periyar E. V. Ramasamy was associated with which

INSTA 75 Days REVISION PLAN for Prelims 2020 - InstaTests

www.insightsonindia.com 31 Insights IAS

(a) 1 only

(b) 2 and 3 only

(c) 1 and 2 only

(d) 1, 2 and 3

Solution: A

Raja Rammohan Roy (1772-1833), often called the father of Indian Renaissance and the

maker of Modern India, was a man of versatile genius.

• Rammohan Roy believed in the modern scientific approach and principles of human

dignity and social equality.

• He put his faith in monotheism. He wrote Gift to Monotheists (1809) and translated

into Bengali the Vedas and the five Upanishads to prove his conviction that ancient

Hindu texts support monotheism.

• In 1814, he set up the Atmiya Sabha (or Society of Friends) in Calcutta to propagate

the monotheistic ideals of the Vedanta and to campaign against idolatry, caste

rigidities, meaningless rituals and other social ills.

• Raja Rammohan Roy founded the Brahmo Sabha in August 1828; it was later renamed

Brahmo Samaj. Through the Sabha he wanted to institutionalise his ideas and mission.

• The long-term agenda of the Brahmo Samaj—to purify Hinduism and to preach

monotheism—was based on the twin pillars of reason and the Vedas and Upanishads.

• Rammohan Roy did not want to establish a new religion. He only wanted to purify

Hinduism of the evil practices which had crept into it. Roy’s progressive ideas met with

strong opposition from orthodox elements like Raja Radhakant Deb who organised

the Dharma Sabha to counter Brahmo Samaj propaganda. Roy’s death in 1833 was a

setback for the Samaj’s mission.

35. Consider the following statements regarding Balshastri Jambhekar

1. He established an orthodox society and stood for the preservation of the status

quo in socio-religious matters.

2. He started the newspaper Darpan.

Which of the statements given above is/are correct?

(a) 1 only

(b) 2 only

(c) Both 1 and 2

(d) Neither 1 nor 2

Page 34: SIMPLYFYING IAS EXAM PREPARATION...• He also founded the organisation Shri Mathura Vrindavan Hasanand Gochar Bhoomi in Vrindavan. 5. Periyar E. V. Ramasamy was associated with which

INSTA 75 Days REVISION PLAN for Prelims 2020 - InstaTests

www.insightsonindia.com 32 Insights IAS

Solution: B

Balshastri Jambhekar (1812-1846) was a pioneer of social reform through journalism in

Bombay; he attacked Brahminical orthodoxy and tried to reform popular Hinduism. He

started the newspaper Darpan in 1832.

• Known as the father of Marathi journalism, Jambhekar used the Darpan to awaken

the people to awareness of social reforms, such as widow remarriage, and to instill in

the masses a scientific approach to life.

• In 1840, he started Digdarshan which published articles on scientific subjects as well

as history.

• Jambhekar founded the Bombay Native General Library and started the Native

Improvement Society of which an offshoot was the Students Literary and Scientific

Library. He was the first professor of Hindi at the Elphinston College, besides being a

director of the Colaba Observatory.

36. Consider the following statements regarding The Theosophical Movement

1. Madame H.P. Blavatsky and Colonel M.S. Olcott founded the Theosophical Society

in Adyar

2. The society sought to investigate the unexplained laws of nature and the powers

latent in man.

3. It accepted the Hindu beliefs in reincarnation and karma and drew inspiration

from the philosophy of the Upanishads.

Which of the statements given above is/are correct?

(a) 1 and 2 only

(b) 2 and 3 only

(c) 1 and 3 only

(d) 1, 2 and 3

Solution: B

The Theosophical Movement

• A group of westerners led by Madame H.P. Blavatsky (1831-1891) and Colonel M.S.

Olcott, who were inspired by Indian thought and culture, founded the Theosophical

Society in New York City, United States in 1875.

Page 35: SIMPLYFYING IAS EXAM PREPARATION...• He also founded the organisation Shri Mathura Vrindavan Hasanand Gochar Bhoomi in Vrindavan. 5. Periyar E. V. Ramasamy was associated with which

INSTA 75 Days REVISION PLAN for Prelims 2020 - InstaTests

www.insightsonindia.com 33 Insights IAS

• In 1882, they shifted their headquarters to Adyar, on the outskirts of Madras (at that

time) in India. The society believed that a special relationship could be established

between a person’s soul and God by contemplation, prayer, revelation, etc.

• It accepted the Hindu beliefs in reincarnation and karma, and drew inspiration from

the philosophy of the Upanishads and samkhya, yoga and Vedanta schools of

thought. It aimed to work for universal brotherhood of humanity without distinction

of race, creed, sex, caste or colour.

• The society also sought to investigate the unexplained laws of nature and the powers

latent in man. The Theosophical Movement came to be allied with the Hindu

renaissance. It opposed child marriage and advocated the abolition of caste

discrimination, uplift of outcastes, improvement in the condition of widows.

• In India, the movement became somewhat popular with the election of Annie Besant

(1847-1933) as its president after the death of Olcott in 1907. Annie Besant had come

to India in 1893.

• She laid the foundation of the Central Hindu College in Benaras in 1898 where both

Hindu religion and Western scientific subjects were taught. The college became the

nucleus for the formation of Benaras Hindu University in 1916. Annie Besant also did

much for the cause of the education of women.

37. Consider the following statements regarding Collegium system

1. It is the system of appointment and transfer of judges that has evolved through

multiple amendments to the constitution.

2. Judges of the higher judiciary are appointed only through the collegium system

and the government has a role only after names have been decided by the

collegium.

Which of the statements given above is/are correct?

(a) 1 only

(b) 2 only

(c) Both 1 and 2

(d) Neither 1 nor 2

Solution: B

• It is the system of appointment and transfer of judges that has evolved through

judgments of the Supreme Court. It is not by an Act of Parliament or by a provision of

the Constitution.

• The Supreme Court collegium is headed by the Chief Justice of India and comprises

four other senior most judges of the court. The collegium system has its genesis in a

Page 36: SIMPLYFYING IAS EXAM PREPARATION...• He also founded the organisation Shri Mathura Vrindavan Hasanand Gochar Bhoomi in Vrindavan. 5. Periyar E. V. Ramasamy was associated with which

INSTA 75 Days REVISION PLAN for Prelims 2020 - InstaTests

www.insightsonindia.com 34 Insights IAS

series of judgments through interpretations of pertinent constitutional provisions

called “Judges Cases”.

Government’s Role

• Judges of the higher judiciary are appointed only through the collegium system and

the government has a role only after names have been decided by the collegium.

• It can also raise objections and seek clarifications regarding the collegium’s choices,

but if the collegium reiterates the same names, the government is bound, under

Constitution Bench judgments, to appoint them as judges

38. Consider the following statements

1. Election commission can take any action it deems fit to ensure that elections and

the election process are free and fair.

2. Model code of conduct does not have statutory value and it is enforced by the

moral and constitutional authority of the Election commission.

Which of the statements given above is/are correct?

(a) 1 only

(b) 2 only

(c) Both 1 and 2

(d) Neither 1 nor 2

Solution: C

• The Election Commission of India is directly established by Article 324 of the

Constitution. It vests in the Commission the superintendence, direction and control of

all elections to Parliament, the State legislatures, and the offices of the President and

Vice-President.

• The EC can take any action it deems fit to ensure that elections and the election

process are free and fair.

• The Model Code of Conduct is a set of guidelines issued by the Election Commission

(EC) to regulate political parties and candidates for the conduct of free and fair

elections. The code comes into force on the announcement of the poll schedule and

remains operational till the process is concluded.

• It does not have statutory value, and it is enforced only by the moral and constitutional

authority of the EC. It is normally legally enforced by invoking other statutes such as

the Indian Penal Code, 1860, Code of Criminal Procedure, 1973, and Representation

of the People Act, 1951.

Page 37: SIMPLYFYING IAS EXAM PREPARATION...• He also founded the organisation Shri Mathura Vrindavan Hasanand Gochar Bhoomi in Vrindavan. 5. Periyar E. V. Ramasamy was associated with which

INSTA 75 Days REVISION PLAN for Prelims 2020 - InstaTests

www.insightsonindia.com 35 Insights IAS

39. Consider the following statements regarding Deccan Riots

1. The ryots of Deccan region of western India suffered heavy taxation under the

Ryotwari system.

2. A social boycott movement organized by the ryots against the outsider

moneylenders.

3. The Deccan Agriculturists Relief Act was passed by Government as a conciliatory

measure.

Which of the statements given above is/are correct?

(a) 1 and 2 only

(b) 2 and 3 only

(c) 1 and 3 only

(d) 1, 2 and 3

Solution: D

• The ryots of Deccan region of western India suffered heavy taxation under the

Ryotwari system.

• Here again the peasants found themselves trapped in a vicious network with the

moneylender as the exploiter and the main beneficiary. These moneylenders were

mostly outsiders—Marwaris or Gujaratis.

• The conditions had worsened due to a crash in cotton prices after the end of the

American Civil War in 1864, the Government’s decision to raise the land revenue by

50% in 1867, and a succession of bad harvests.

• In 1874, the growing tension between the moneylenders and the peasants resulted in

a social boycott movement organized by the ryots against the “outsider”

moneylenders.

• The ryots refused to buy from their shops. No peasant would cultivate their fields. The

barbers, washermen, shoemakers would not serve them. This social boycott spread

rapidly to the villages of Poona, Ahmednagar, Sholapur and Satara. Soon the social

boycott was transformed into agrarian riots with systematic attacks on the

moneylenders’ houses and shops.

• The debt bonds and deeds were seized and publicly burnt. The Government

succeeded in repressing the movement.

• As a conciliatory measure, the Deccan Agriculturists Relief Act was passed in 1879.

This time also, the modern nationalist intelligentsia of Maharashtra supported the

peasants’ cause.

Page 38: SIMPLYFYING IAS EXAM PREPARATION...• He also founded the organisation Shri Mathura Vrindavan Hasanand Gochar Bhoomi in Vrindavan. 5. Periyar E. V. Ramasamy was associated with which

INSTA 75 Days REVISION PLAN for Prelims 2020 - InstaTests

www.insightsonindia.com 36 Insights IAS

40. Which of the following vows were taken under Eka or the Unity Movement

1. Pay only the recorded rent but would pay it on time

2. Give no help to criminals

3. Refuse to do forced labour

4. Abide by panchayat decisions

Which of the statements given above is/are correct?

(a) 1, 2 and 3 only

(b) 2, 3 and 4 only

(c) 1, 3 and 4 only

(d) 1, 2, 3 and 4

Solution: D

Eka Movement

Towards the end of 1921, peasant discontent resurfaced in some northern districts of the

United Provinces—Hardoi, Bahraich, Sitapur. The issues involved were:

1. high rents—50 per cent higher than the recorded rates;

2. oppression of thikadars in charge of revenue collection; and

3. practice of share-rents.

The meetings of the Eka or the Unity Movement involved a symbolic religious ritual in which

the assembled peasants vowed that they would

1. pay only the recorded rent but would pay it on time;

2. not leave when evicted;

3. refuse to do forced labour;

4. give no help to criminals;

5. abide by panchayat decisions.

The grassroot leadership of the Eka Movement came from Madari Pasi and other low-caste

leaders, and many small zamindars.

By March 1922, severe repression by authorities brought the movement to an end.

41. Consider the following statements regarding Petroleum & Explosives Safety

Organization

1. It functions under the Ministry of Petroleum and Natural Gas

Page 39: SIMPLYFYING IAS EXAM PREPARATION...• He also founded the organisation Shri Mathura Vrindavan Hasanand Gochar Bhoomi in Vrindavan. 5. Periyar E. V. Ramasamy was associated with which

INSTA 75 Days REVISION PLAN for Prelims 2020 - InstaTests

www.insightsonindia.com 37 Insights IAS

2. It is the apex department to control and administer manufacture, storage,

transport and handling of explosives, petroleum, compressed gases and other

hazardous substances in India.

Which of the statements given above is/are correct?

(a) 1 only

(b) 2 only

(c) Both 1 and 2

(d) Neither 1 nor 2

Solution: B

Petroleum & Explosives Safety Organization:

• It is a department under Department for the Promotion of Industry and Internal

Trade under Ministry of Commerce and Industry.

• It is a regulatory authority with autonomous status.

• It was established during the British India in 1890s as Department of Explosives and

later expanded to various other activities.

• As a statutory authority, PESO is entrusted with the responsibilities under the

Explosives Act, 1884; Petroleum Act, 1934; Inflammable Substances Act, 1952,

Environment (Protection Act), 1986.

• Why in News? Petroleum & Explosives Safety Organization takes various measures to

address the problems faced by Petroleum, Explosives, Oxygen and Industrial Gas

Industries.

42. Consider the following statements regarding Kyasanoor Forest Disease (KFD)

1. It is endemic to the Karnataka state.

2. It can transmit through person’s saliva and body fluid.

3. There is no vaccine for KFD.

Which of the statements given above is/are correct?

(a) 1 and 2 only

(b) 1 only

(c) 1 and 3 only

(d) 1, 2 and 3

Page 40: SIMPLYFYING IAS EXAM PREPARATION...• He also founded the organisation Shri Mathura Vrindavan Hasanand Gochar Bhoomi in Vrindavan. 5. Periyar E. V. Ramasamy was associated with which

INSTA 75 Days REVISION PLAN for Prelims 2020 - InstaTests

www.insightsonindia.com 38 Insights IAS

Solution: B

Kyasanur Forest Disease

• KFD is caused by the Kyasanur Forest Disease Virus (KFDV). The virus was identified

in 1957 when it was isolated from a sick monkey from the Kyasanur Forest. Since then,

between 400-500 humans cases per year have been reported.

• Hard ticks (Hemaphysalis spinigera) are the reservoir of the KFD virus and once

infected, remain so for life.

• Rodents, shrews, and monkeys are common hosts for KFDV after being bitten by an

infected tick. KFDV can cause epizootics with high fatality in primates.

• KFD is endemic to the Indian state of Karnataka.

• The virus is transmitted to human beings through parasitic ticks which latch on to

monkeys. No person-to-person transmission has been described.

• A vaccine does exist for KFD and is used in endemic areas of India.

43. Consider the following statements regarding All India Trade Union Congress

1. The All India Trade Union Congress was founded in 1920 by Lala Lajpat Rai.

2. The AITUC was influenced by social democratic ideas of the British Labour Party

and Gandhian philosophy.

Which of the statements given above is/are correct?

(a) 1 only

(b) 2 only

(c) Both 1 and 2

(d) Neither 1 nor 2

Solution: B

• A follower of Gokhale, Narayan Malhar Joshi founded the Social Service League in

Bombay with an aim to secure for the masses better and reasonable conditions of life

and work. Joshi also founded the All India Trade Union Congress (1920).

• The All India Trade Union Congress was founded on October 31, 1920. The Indian

National Congress president for the year, Lala Lajpat Rai, was elected as the first

president of AITUC and Dewan Chaman Lal as the first general secretary.

• Lajpat Rai was the first to link capitalism with imperialism—“imperialism and

militarism are the twin children of capitalism”.

Page 41: SIMPLYFYING IAS EXAM PREPARATION...• He also founded the organisation Shri Mathura Vrindavan Hasanand Gochar Bhoomi in Vrindavan. 5. Periyar E. V. Ramasamy was associated with which

INSTA 75 Days REVISION PLAN for Prelims 2020 - InstaTests

www.insightsonindia.com 39 Insights IAS

• The prominent Congress and swarajist leader C.R. Das presided over the third and the

fourth sessions of the AITUC.

• The Gaya session of the Congress (1922) welcomed the formation of the AITUC and

a committee was formed to assist it. C.R. Das advocated that the Congress should take

up the workers’ and peasants’ cause and incorporate them in the struggle for swaraj

or else they would get isolated from the movement. Other leaders who kept close

contacts with the AITUC included Nehru, Subhas Bose, C.F. Andrews, J.M. Sengupta,

Satyamurthy, V.V. Giri and Sarojini Naidu. In the beginning, the AITUC was influenced

by social democratic ideas of the British Labour Party. The Gandhian philosophy of

non-violence, trusteeship and class-collaboration had great influence on the

movement. Gandhi helped organize the Ahmedabad Textile Labour Association

(1918) and through a protest secured a 27.5 per cent wage hike.

44. Which of the following are the features of The Trade Union Act, 1926

1. Recognized trade unions as legal associations

2. Secured immunity, both civil and criminal, for trade unions from prosecution for

legitimate activities

3. It made the strikes in public utility services illegal.

Which of the statements given above is/are correct?

(a) 1 and 2 only

(b) 2 and 3 only

(c) 1 and 3 only

(d) 1, 2 and 3

Solution: A

The Trade Union Act, 1926

• recognised trade unions as legal associations;

• laid down conditions for registration and regulation of trade union activities;

• secured immunity, both civil and criminal, for trade unions from prosecution for

legitimate activities, but put some restrictions on their political activities.

The Trade Disputes Act (TDA), 1929

• made compulsory the appointment of Courts of Inquiry and Consultation Boards for

settling industrial disputes;

Page 42: SIMPLYFYING IAS EXAM PREPARATION...• He also founded the organisation Shri Mathura Vrindavan Hasanand Gochar Bhoomi in Vrindavan. 5. Periyar E. V. Ramasamy was associated with which

INSTA 75 Days REVISION PLAN for Prelims 2020 - InstaTests

www.insightsonindia.com 40 Insights IAS

• made illegal the strikes in public utility services like posts, railways, water and

electricity, unless each individual worker planning to go on strike gave an advance

notice of one month to the administration;

• forbade trade union activity of coercive or purely political nature and even

sympathetic strikes.

45. Nair movement was led by

(a) Shri Narayana Guru

(b) Dr T.M. Nair, P. Chetti and C.N. Mudalair

(c) E.V. Ramaswami Naiker

(d) C.V. Raman Pillai, K. Rama Pillai, and M. Padmanabha Pillai

Solution: D

• Aravippuram Movement led by Shri Narayana Guru

• Justice party Movement led by Dr T.M. Nair, P. Chetti and C.N. Mudalair on behalf of

castes

• Nair Movement led by C.V. Raman Pillai, K. Rama Pillai, and M. Padmanabha Pillai

• Self-respect Movement was led by E.V. Ramaswami Naiker

46. Consider the following statements regarding Appropriation Bill

1. The Rajya Sabha doesn’t have the power to recommend any amendments in this

Bill.

2. Once the Appropriation Bill has been passed, only President can propose

amendments.

Which of the statements given above is/are correct?

(a) 1 only

(b) 2 only

(c) Both 1 and 2

(d) Neither 1 nor 2

Solution: D

Page 43: SIMPLYFYING IAS EXAM PREPARATION...• He also founded the organisation Shri Mathura Vrindavan Hasanand Gochar Bhoomi in Vrindavan. 5. Periyar E. V. Ramasamy was associated with which

INSTA 75 Days REVISION PLAN for Prelims 2020 - InstaTests

www.insightsonindia.com 41 Insights IAS

Appropriation Bill:

• Appropriation Bill is a money bill that allows the government to withdraw funds from

the Consolidated Fund of India to meet its expenses during the course of a financial

year.

• As per article 114 of the Constitution, the government can withdraw money from the

Consolidated Fund only after receiving approval from Parliament.

• To put it simply, the Finance Bill contains provisions on financing the expenditure of

the government, and Appropriation Bill specifies the quantum and purpose for

withdrawing money.

Procedure followed:

• The government introduces the Appropriation Bill in the lower house of Parliament

after discussions on Budget proposals and Voting on Demand for Grants.

• The Appropriation Bill is first passed by the Lok Sabha and then sent to the Rajya

Sabha.

• The Rajya Sabha has the power to recommend any amendments in this Bill. However,

it is the prerogative of the Lok Sabha to either accept or reject the recommendations

made by the upper house of Parliament.

• The unique feature of the Appropriation Bill is its automatic repeal clause, whereby

the Act gets repealed by itself after it meets its statutory purpose.

• Once the Appropriation Bill has been passed, no amendments in its amounts can be

proposed in either Parliament or legislature.

47. Consider the following statements regarding The Pagal Panthis

1. They are semi-religious group mainly constituting the Hajong and Garo tribes.

2. They refused to pay rent above a certain limit and attacked the houses of

zamindars.

Which of the statements given above is/are correct?

(a) 1 only

(b) 2 only

(c) Both 1 and 2

(d) Neither 1 nor 2

Solution: C

Page 44: SIMPLYFYING IAS EXAM PREPARATION...• He also founded the organisation Shri Mathura Vrindavan Hasanand Gochar Bhoomi in Vrindavan. 5. Periyar E. V. Ramasamy was associated with which

INSTA 75 Days REVISION PLAN for Prelims 2020 - InstaTests

www.insightsonindia.com 42 Insights IAS

The Pagal Panthis

• The Pagal Panthi, a semi-religious group mainly constituting the Hajong and Garo

tribes of Mymensingh district (earlier in Bengal), was founded by Karam Shah. But the

tribal peasants organised themselves under Karam Shah’s son, Tipu, to fight the

oppression of the zamindars.

• From 1825 to 1835, the Pagal Panthis refused to pay rent above a certain limit and

attacked the houses of zamindars. The government introduced an equitable

arrangement to protect these peasants, but the movement was violently suppressed.

48. Consider the following statements regarding The Servants of India Society

1. Gopal Krishna Gokhale founded the Servants of India Society.

2. The aim of the society was to train national missionaries for the service of India

3. The Hitavada was published to project the views of the society.

Which of the statements given above is/are correct?

(a) 1 and 2 only

(b) 2 and 3 only

(c) 1 and 3 only

(d) 1, 2 and 3

Solution: D

The Servants of India Society

• Gopal Krishna Gokhale (1866-1915), a liberal leader of the Indian National Congress,

founded the Servants of India Society in 1905 with the help of M.G. Ranade.

• The aim of the society was to train national missionaries for the service of India; to

promote, by all constitutional means, the true interests of the Indian people; and to

prepare a cadre of selfless workers who were to devote their lives to the cause of the

country in a religious spirit.

• In 1911, the Hitavada began to be published to project the views of the society.

• The society chose to remain aloof from political activities and organizations like the

Indian National Congress.

• After Gokhale’s death (1915), Srinivasa Shastri took over as president. The society

still continues to function, though with a shrunken base, at many places in India. It

works in the field of education, providing ashram type of schools for tribal girls and

balwadis at many places.

Page 45: SIMPLYFYING IAS EXAM PREPARATION...• He also founded the organisation Shri Mathura Vrindavan Hasanand Gochar Bhoomi in Vrindavan. 5. Periyar E. V. Ramasamy was associated with which

INSTA 75 Days REVISION PLAN for Prelims 2020 - InstaTests

www.insightsonindia.com 43 Insights IAS

49. Consider the following statements regarding International Maritime Organisation (IMO)

1. It is a United Nations specialized agency with responsibility for the safety and

security of shipping and the prevention of marine pollution by ships.

2. India has not ratified the treaty.

Which of the statements given above is/are correct?

(a) 1 only

(b) 2 only

(c) Both 1 and 2

(d) Neither 1 nor 2

Solution: A

• International Maritime Organisation (IMO) is a United Nations specialized agency

with responsibility for the safety and security of shipping and the prevention of marine

pollution by ships.

• Its main role is to create a regulatory framework for the shipping industry that is fair

and effective, universally adopted and universally implemented.

• India has been one of the earliest members of the IMO, having ratified its Convention

and joined it as a member-state in the year 1959.

• India has had the privilege of being elected to and serving the Council of the IMO, ever

since it started functioning, and till date, except for two years for the period 1983-

1984.

• India is a party to 34 IMO Conventions and protocols and is currently in the advanced

stage of ratifying Ballast Water Convention and Bunker Convention.

Extra Reading

IMO Council

• The IMO Council consists of 40 member countries.

• In Categories A” and B” there are 10 members each and in Category C” 20 members,

who are elected by the IMO Assembly.

• IMO Council plays a crucial role to play in deciding various important matters within

the mandate of the IMO, in relation to the global shipping industry, including its work

programme strategy and budget.

• Ballast Water Convention and Bunker Convention.

Page 46: SIMPLYFYING IAS EXAM PREPARATION...• He also founded the organisation Shri Mathura Vrindavan Hasanand Gochar Bhoomi in Vrindavan. 5. Periyar E. V. Ramasamy was associated with which

INSTA 75 Days REVISION PLAN for Prelims 2020 - InstaTests

www.insightsonindia.com 44 Insights IAS

50. Consider the following statements regarding Event Horizon Telescope (EHT)

1. EHT project was devised by NASA in 2012 to directly observe the immediate

environment of a black hole.

2. It is a network of 10 radio telescopes on four continents that collectively operate

like a single instrument nearly the size of the Earth.

Which of the statements given above is/are correct?

(a) 1 only

(b) 2 only

(c) Both 1 and 2

(d) Neither 1 nor 2

Solution: B

• The EHT is a project of network of 10 radio telescopes on four continents that

collectively operate like a single instrument nearly the size of the Earth.

• The EHT project, an international partnership formed in 2012 is to directly observe

the immediate environment of a black hole.

• Its aim was to capture the first image of a black hole by creating a virtual Earth-sized

telescope.

• It is a project to create a large telescope array consisting of a global network of radio

telescopes and combining data from several verylong-baseline interferometry (VLBI)

stations around the Earth

• This technique of linking radio dishes across the globe to create an Earth-sized

interferometer has been used to measure the size of the emission regions of the two

supermassive black holes.

https://eventhorizontelescope.org/about

DAY – 35 (InstaTest-35)

51. Consider the following statements

1. Sagittarius A* is believed to be a super massive galaxy present in the Universe.

2. The Kuiper belt a circumstellar disc in the outer Solar System, extending from the

orbit of Neptune to approximately 50 AU from the Sun.

Which of the statements given above is/are correct?

(a) 1 only

Page 47: SIMPLYFYING IAS EXAM PREPARATION...• He also founded the organisation Shri Mathura Vrindavan Hasanand Gochar Bhoomi in Vrindavan. 5. Periyar E. V. Ramasamy was associated with which

INSTA 75 Days REVISION PLAN for Prelims 2020 - InstaTests

www.insightsonindia.com 45 Insights IAS

(b) 2 only

(c) Both 1 and 2

(d) Neither 1 nor 2

Solution: B

• Sagittarius A or Sgr A is a complex radio source at the center of the Milky Way which

contains a supermassive black hole. It is located in the constellation Sagittarius, and

is hidden from view at optical wavelengths by large clouds of cosmic dust in the spiral

arms of the Milky Way.

• Kuiper Belt — a donut-shaped region of icy bodies beyond the orbit of Neptune. There

may be millions of these icy objects, collectively referred to as Kuiper Belt objects

(KBOs) or trans-Neptunian objects (TNOs), in this distant region of our solar system.

It is extending from the orbit of Neptune to approximately 50 AU from the Sun.

https://www.thehindu.com/sci-tech/science/watch-all-about-black-

holes/article29446293.ece

52. Consider the following statements

1. Treaty of Sagauli was signed following First Burma War.

2. Treaty of Yandabo was signed between British and Gorkhas.

Page 48: SIMPLYFYING IAS EXAM PREPARATION...• He also founded the organisation Shri Mathura Vrindavan Hasanand Gochar Bhoomi in Vrindavan. 5. Periyar E. V. Ramasamy was associated with which

INSTA 75 Days REVISION PLAN for Prelims 2020 - InstaTests

www.insightsonindia.com 46 Insights IAS

Which of the statements given above is/are correct?

(a) 1 only

(b) 2 only

(c) Both 1 and 2

(d) Neither 1 nor 2

Solution: D

Anglo-Nepalese Relations

• The Gorkhas wrested control of Nepal from the successors of Ranjit Malla of

Bhatgaon in 1760. They began to expand their dominion beyond the mountains. They

found it easier to expand in the southern direction, as the north was well defended by

the Chinese.

• In 1801, the English annexed Gorakhpur which brought the Gorkhas’ boundary and

the Company’s boundary together. The conflict started due to the Gorkhas’ capture

of Butwal and Sheoraj in the period of Lord Hastings (1813-23). The war, ended in the

Treaty of Sagauli, 1816 which was in favour of the British.

First Burma War (1824-26)

• The first war with Burma was fought when the Burmese expansion westwards and

occupation of Arakan and Manipur, and the threat to Assam and the Brahmaputra

Valley led to continuous friction along the ill-defined border between Bengal and

Burma, in the opening decades of the nineteenth century. The British expeditionary

forces occupied Rangoon in May 1824 and reached within 72 km of the capital at Ava.

Peace was established in 1826 with the Treaty of Yandabo.

53. Consider the following pairs regarding foreign policy with Afghanistan

1. Forward Policy : Auckland 2. Policy of Masterly Inactivity : John Lawrence 3. Policy of Proud Reserve : Lytton

Which of the pairs given above is/are matched correctly?

(a) 3 only

(b) 1 and 2 only

(c) 2 and 3 only

(d) 1, 2 and 3

Page 49: SIMPLYFYING IAS EXAM PREPARATION...• He also founded the organisation Shri Mathura Vrindavan Hasanand Gochar Bhoomi in Vrindavan. 5. Periyar E. V. Ramasamy was associated with which

INSTA 75 Days REVISION PLAN for Prelims 2020 - InstaTests

www.insightsonindia.com 47 Insights IAS

Solution: D

Forward Policy of Auckland

• Auckland who came to India as the governor-general in 1836, advocated a forward

policy. This implied that the Company government in India itself had to take initiatives

to protect the boundary of British India from a probable Russian attack.

• This objective was to be achieved either through treaties with the neighbouring

countries or by annexing them completely.

John Lawrence and the Policy of Masterly Inactivity

• John Lawrence (1864-1869) started a policy of masterly inactivity which was a

reaction to the disasters of the First Afghan War and an outcome of practical common

sense and an intimate knowledge of the frontier problem and of Afghan passion for

independence.

Lytton and the Policy of Proud Reserve

• Lytton, a nominee of the Conservative government under Benjamin Disraeli (1874-

80), became the Viceroy of India in 1876. He started a new foreign policy of ‘proud

reserve’, which was aimed at having scientific frontiers and safeguarding ‘spheres of

influence’. According to Lytton, the relations with Afghanistan could no longer be left

ambiguous.

54. Which of the following rights were guaranteed under Karachi Resolution on

Fundamental Rights

1. Free speech and free press

2. Free and compulsory primary education

3. Universal adult franchise

4. Equal legal rights irrespective of caste, creed and sex

Select the correct answer using the code given below

(a) 1, 2 and 3 only

(b) 2, 3 and 4 only

(c) 1, 3 and 4 only

(d) 1, 2, 3 and 4

Solution: D

Page 50: SIMPLYFYING IAS EXAM PREPARATION...• He also founded the organisation Shri Mathura Vrindavan Hasanand Gochar Bhoomi in Vrindavan. 5. Periyar E. V. Ramasamy was associated with which

INSTA 75 Days REVISION PLAN for Prelims 2020 - InstaTests

www.insightsonindia.com 48 Insights IAS

Karachi Congress Session—1931

• In March 1931, a special session of the Congress was held at Karachi to endorse the

Gandhi-Irwin Pact. Six days before the session (which was held on March 29) Bhagat

Singh, Sukhdev and Rajguru were executed. Throughout Gandhi’s route to Karachi,

he was greeted with black flag demonstrations by the Punjab Naujawan Bharat

Sabha, in protest against his failure to secure commutation of the death sentence for

Bhagat and his comrades.

Congress Resolutions at Karachi

• While disapproving of and dissociating itself from political violence, the Congress

admired the ‘bravery’ and ‘sacrifice’ of the three martyrs.

• The Delhi Pact or Gandhi-Irwin Pact was endorsed.

• The goal of purna swaraj was reiterated.

• Two resolutions were adopted—one on Fundamental Rights and the other on

National Economic Programme which made the session particularly memorable.

The Resolution on Fundamental Rights guaranteed—

1. free speech and free press

2. right to form associations

3. right to assemble

4. universal adult franchise

5. equal legal rights irrespective of caste, creed and sex

6. neutrality of state in religious matters

7. free and compulsory primary education

8. protection to culture, language, script of minorities and linguistic groups

55. Consider the following statements regarding Haripura session

1. This was the session held in a village for the first time.

2. Subhash Chandra Bose was unanimously elected president of the session.

3. National Planning Committee was set up under the chairmanship of Jawaharlal

Nehru.

Which of the statements given above is/are correct?

(a) 1 and 2 only

(b) 2 and 3 only

(c) 1 and 3 only

(d) 1, 2 and 3

Solution: B

Page 51: SIMPLYFYING IAS EXAM PREPARATION...• He also founded the organisation Shri Mathura Vrindavan Hasanand Gochar Bhoomi in Vrindavan. 5. Periyar E. V. Ramasamy was associated with which

INSTA 75 Days REVISION PLAN for Prelims 2020 - InstaTests

www.insightsonindia.com 49 Insights IAS

Haripura

• At the Congress meeting in Haripura, Gujarat, in February 1938, Bose was

unanimously elected president of the session. He was firm in his belief that the

Congress ministries in the provinces had immense revolutionary potential, as he said

in his presidential address. Bose also talked of economic development of the country

through planning and was instrumental in setting up a National Planning Committee

later.

• The session adopted a resolution that the Congress would give moral support to those

who were agitating against the governance in the princely states.

• In the following months, the international situation was highly disturbed; there were

clear signs that Europe was going to be embroiled in war.

• National Planning Committee was set up under the chairmanship of Jawaharlal

Nehru.

• Faizpur 1936: was the session held in a village for the first time.

56. Consider the following statements regarding Nilgiri Tahr

1. It is endemic to the Nilgiri Hills and Western Ghats

2. It has been listed as Critically Endangered by IUCN.

3. It is found in open montane grassland habitat of rain forests eco region.

Which of the statements given above is/are correct?

(a) 1 and 2 only

(b) 2 and 3 only

(c) 1 and 3 only

(d) 1, 2 and 3

Solution: C

Nilgiri Tahr:

• Recently, Nilgiri tahr’s population has increased from 568 in 2018 to 612 in 2019 in the

Mukurthi National Park, Tamil Nadu.

• Nilgiri Tahr is also known as Nilgiri Ibex.

• State animal of Tamil Nadu

• Only species of Caprine ungulate that is found south of the Himalayas in India

• Habitat: Open montane grassland habitat of the South Western Ghats montane rain

forests Eco region. Endemic to the Nilgiri Hills and the southern portion of the

Western Ghats in the states of Tamil Nadu and Kerala in Southern India (ex: Anamalai

Hills, Palni Hills)

Page 52: SIMPLYFYING IAS EXAM PREPARATION...• He also founded the organisation Shri Mathura Vrindavan Hasanand Gochar Bhoomi in Vrindavan. 5. Periyar E. V. Ramasamy was associated with which

INSTA 75 Days REVISION PLAN for Prelims 2020 - InstaTests

www.insightsonindia.com 50 Insights IAS

• It has been listed as “Endangered” by IUCN.

• It has been listed under Schedule 1 of the Wildlife (Protection) Act, 1972 which

provides absolute protection and offences under these are prescribed the highest

penalties

• The Adult males of Nilgiri Tahr species develop a light grey area or “saddle” on their

backs and are hence called “Saddlebacks”

57. Consider the following statements regarding civil services reforms

1. Cornwallis was the first to bring into existence and organize the civil services.

2. Aitchison Committee decreased the age limit to 21.

Which of the statements given above is/are correct?

(a) 1 only

(b) 2 only

(c) Both 1 and 2

(d) Neither 1 nor 2

Solution: A

Page 53: SIMPLYFYING IAS EXAM PREPARATION...• He also founded the organisation Shri Mathura Vrindavan Hasanand Gochar Bhoomi in Vrindavan. 5. Periyar E. V. Ramasamy was associated with which

INSTA 75 Days REVISION PLAN for Prelims 2020 - InstaTests

www.insightsonindia.com 51 Insights IAS

Cornwallis (governor-general, 1786-93) was the first to bring into existence and organize the

civil services. He tried to check corruption through—

• raising the civil servants’ salary,

• strict enforcement of rules against private trade,

• debarring civil servants from taking presents, bribes etc.,

• enforcing promotions through seniority.

The Aitchison Committee on Public Services (1886), set up by Dufferin, recommended—

• dropping of the terms ‘covenanted’ and ‘uncovenanted’;

• classification of the civil service into Imperial Indian Civil Service (examination in

England), Provincial Civil Service (examination in India) and Subordinate Civil Service

(examination in India); and,

• raising the age limit to 23.

58. Match the following committees with the purpose it was setup for:

1. Hilton Young Commission Labour Reforms 2. Mac Donnell Commission Police Reforms 3. Floud Commission Bengal Land Revenue 4. Butler Commission Indian States relation with British Crown

Which of the pairs given above is/are matched correctly?

(a) 1 and 2 only

(b) 2 and 3 only

(c) 3 and 4 only

(d) 1 and 4 only

Solution: C

1. The Hilton Young Commission was a Commission of Inquiry appointed in 1926 to look

into the possible closer union of the British territories in East and Central Africa. These

were individually economically underdeveloped, and it was suggested that some form

of association would result both in cost savings and their more rapid development.

2. The MacDonnell Commission was the famine commission appointed by Lord Curzon,

after a long period of 20 years, when Lord Lytton had tried to formulate the general

principles of dealing with such famines.

3. Floud Commission a land revenue commission established by the government of

Bengal in 1938

4. The Indian states committee appointed a committee under the Chairmanship of Sir

Harcourt Butler which was popularly known as ‘the Butler Committee’ to investigate

Page 54: SIMPLYFYING IAS EXAM PREPARATION...• He also founded the organisation Shri Mathura Vrindavan Hasanand Gochar Bhoomi in Vrindavan. 5. Periyar E. V. Ramasamy was associated with which

INSTA 75 Days REVISION PLAN for Prelims 2020 - InstaTests

www.insightsonindia.com 52 Insights IAS

and clarify the relationship between the paramount power and the Princes of Princely

States in AD 1927.

Extra Reading:

Famine Commissions

1866 : Campbell Commission

Viceroy : John Lawrence

1880 : Stratchy Commission (Famine Commission)

Viceroy : Lytton

Chairman : Richard Strachey

Objective : Give relief famine sricken

1897 : Lyall Commission (Famine Commission)

Viceroy : Elgin

Chairman : James Lyall

Objective : Suggestion to earlier report

1901 : Mac Donnell Commission (Famine Commission)

Viceroy : Curzon

Chairman : Anthony MacDonnel

Objective : Investigate events of Bengal famine

Currency Commissions

1886 : Mansfield Commission by Dufferin

1898 : Fowler Commission by Elgin II

1919 : Babington Smith Commission by Chelmsford

1926 : Hilton Young Commission by Linlithgow

Page 55: SIMPLYFYING IAS EXAM PREPARATION...• He also founded the organisation Shri Mathura Vrindavan Hasanand Gochar Bhoomi in Vrindavan. 5. Periyar E. V. Ramasamy was associated with which

INSTA 75 Days REVISION PLAN for Prelims 2020 - InstaTests

www.insightsonindia.com 53 Insights IAS

Other Commissions

1901 : Scott-Moncrieff Commission (Irrigation) by Curzon

1902 : Fraser Commission (Police Reforms) by Curzon

1919 : Hunter Commission (Punjab Disturbances) by Chelmsford

1927 : Butler Commission (Indian States relation with British Crown) by Irwin

1929 : Whitelay Commission (Labour) by Irwin

1935 : Sapru Commission (Unemployment) by Linlithgow

1939 : Chalfield Commission (Army) by Linlighgow

1940 : Floud Commission (Bengal Land Revenue) by Linlighgow

59. Consider the following statements regarding Local self-government

1. Financial decentralization was a legislative devolution inaugurated by the Indian

Councils Act of 1861.

2. Lord Mayo is called father of local self-government in India.

Which of the statements given above is/are correct?

(a) 1 only

(b) 2 only

(c) Both 1 and 2

(d) Neither 1 nor 2

Solution: A

Mayo’s Resolution of 1870

• Financial decentralization was a legislative devolution inaugurated by the Indian

Councils Act of 1861. Apart from the annual grant from imperial Government, the

provincial governments were authorized to resort to local taxation to balance their

budgets. This was done in context of transfer of certain departments of

administration, such as medical services, education and roads, to the control of

provincial governments. This was the beginning of local finance. Mayo’s Resolution

emphasized, “Local interest, supervision and care are necessary for success in the

management of the funds devoted to education, sanitation, medical relief and local

public works.”

Page 56: SIMPLYFYING IAS EXAM PREPARATION...• He also founded the organisation Shri Mathura Vrindavan Hasanand Gochar Bhoomi in Vrindavan. 5. Periyar E. V. Ramasamy was associated with which

INSTA 75 Days REVISION PLAN for Prelims 2020 - InstaTests

www.insightsonindia.com 54 Insights IAS

Ripon’s Resolution of 1882

• The Government of Ripon desired the provincial governments to apply in case of local

bodies the same principle of financial decentralization which Lord Mayo’s

Government had begun towards them. For his contributions, Lord Ripon is called

father of local self-government in India.

60. Consider the following statements regarding Mt Aconcagua

1. It is located in Uruguay

2. It is part of the Andes mountain range

Which of the statements given above is/are correct?

(a) 1 only

(b) 2 only

(c) Both 1 and 2

(d) Neither 1 nor 2

Solution: B

Page 57: SIMPLYFYING IAS EXAM PREPARATION...• He also founded the organisation Shri Mathura Vrindavan Hasanand Gochar Bhoomi in Vrindavan. 5. Periyar E. V. Ramasamy was associated with which

INSTA 75 Days REVISION PLAN for Prelims 2020 - InstaTests

www.insightsonindia.com 55 Insights IAS

• A 12-year old Mumbai student, Kaamya Karthikeyan has set a record of becoming the

youngest in the world to summit Mt. Aconcagua.

About Mt. Aconcagua:

• It is the highest peak of the Andes Mountains in Argentina, South America.

• It is the highest mountain outside of Asia, with a summit elevation of 6,960.8 metres.

• The mountain is one of the so-called Seven Summits of the seven continents.

• It is Part of the Andes mountain range in South America.

61. Consider the following statements regarding National Organic food Festival

1. It will be jointly organized by Ministry of Food Processing Industries and Ministry

of Women and Child Development.

2. It aims to strengthen the organic market and empower women entrepreneurs in

the area of production and processing of organic products.

Which of the statements given above is/are correct?

(a) 1 only

(b) 2 only

(c) Both 1 and 2

(d) Neither 1 nor 2

Solution: C

National Organic food Festival

• It will be jointly organized by Ministry of Food Processing Industries (MoFPI) and

Ministry of Women and Child Development (Mo WCD).

• Theme of the festival is Unleashing India’s Organic Market Potential.

• The event will focus on facilitating business linkages and empowering women

entrepreneurs through pre-arranged B2B and B2G meetings.

• It aims to strengthen the organic market and empower women entrepreneurs in the

area of production and processing of organic products.

• Women Entrepreneurs and Self Help groups (SHG’s) from all over the country will be

exhibiting their organic products in various segments such as fruit & vegetables, ready

to eat products, spices and condiments, honey, cereals, dry fruits etc.

Page 58: SIMPLYFYING IAS EXAM PREPARATION...• He also founded the organisation Shri Mathura Vrindavan Hasanand Gochar Bhoomi in Vrindavan. 5. Periyar E. V. Ramasamy was associated with which

INSTA 75 Days REVISION PLAN for Prelims 2020 - InstaTests

www.insightsonindia.com 56 Insights IAS

62. Which of the following was/were the provisions of Indian Factory Act, 1891

1. Employment of children under 7 years of age prohibited

2. It fixed maximum working hours for women at 11 hours per day.

3. It provided weekly holiday for all.

Select the correct answer using the code given below

(a) 1 only

(b) 2 and 3 only

(c) 1 and 2 only

(d) 1, 2 and 3

Solution: B

The Indian Factory Act, 1881 dealt primarily with the problem of child labour (between 7 and

12 years of age).

Its significant provisions were:

• employment of children under 7 years of age prohibited,

• working hours restricted to 9 hours per day for children,

• children to get four holidays in a month,

• hazardous machinery to be properly fenced off.

The Indian Factory Act, 1891

1. increased the minimum age (from 7 to 9 years) and the maximum (from 12 to 14

years) for children,

2. reduced maximum working hours for children to 7 hours a day,

3. fixed maximum working hours for women at 11 hours per day with an one-and-a-half

hour interval (working hours for men were left unregulated),

4. provided weekly holiday for all.

63. Consider the following statements regarding Siddi Tribes

1. They are originated from Africa.

2. They are largely living in Karnataka, Andhra Pradesh, Gujarat and Maharashtra

states.

3. They are included in the Particularly Vulnerable Tribal Groups.

Which of the statements given above is/are correct?

(a) 1 and 3 only

Page 59: SIMPLYFYING IAS EXAM PREPARATION...• He also founded the organisation Shri Mathura Vrindavan Hasanand Gochar Bhoomi in Vrindavan. 5. Periyar E. V. Ramasamy was associated with which

INSTA 75 Days REVISION PLAN for Prelims 2020 - InstaTests

www.insightsonindia.com 57 Insights IAS

(b) 2 only

(c) 3 only

(d) 1, 2 and 3

Solution: A

• The Siddi community is also known by different synonyms such as Habshi and Badsha.

It is believed that they are of African origin because they clearly show the Negroid

racial strain in their physical features. Descendants of Bantu people of East Africa,

Siddi ancestors were largely brought to India as slaves by Arabs as early as the 7th

Century, followed by the Portuguese and the British later on.

• At present, the Siddis are living on the western coast of Gujarat, Maharashtra and

Karnataka states. In Karnataka, they mainly live in Dharwad, Belagavi and Uttar

Kannada districts.

• In India, the Union government in 2003, classified Siddis under the list of Scheduled

Tribes. They are included in the Centre’s list of Particularly Vulnerable Tribal Groups

64. Who among the following was/were Economic critic of British rule?

1. Dadabhai Naoroji

2. Romesh Chandra Dutt

3. Gopal Krishna Gokhale

4. Prithwishchandra Ray

Select the correct answer using the code given below

(a) 1, 2 and 3 only

(b) 2, 3 and 4 only

(c) 1, 3 and 4 only

(d) 1, 2, 3 and 4

Solution: D

• Dadabhai Naoroji, the ‘Grand Old Man of India’, who after a brilliant analysis of the

colonial economy put forward the theory of economic drain in Poverty and UnBritish

Rule in India.

Page 60: SIMPLYFYING IAS EXAM PREPARATION...• He also founded the organisation Shri Mathura Vrindavan Hasanand Gochar Bhoomi in Vrindavan. 5. Periyar E. V. Ramasamy was associated with which

INSTA 75 Days REVISION PLAN for Prelims 2020 - InstaTests

www.insightsonindia.com 58 Insights IAS

• Other economic analysts included Justice Mahadeo Govind Ranade, Romesh Chandra

Dutt (The Economic History of India), Gopal Krishna Gokhale, G. Subramaniya Iyer

and Prithwishchandra Ray.

• The essence of nineteenth century colonialism, they said, lay in the transformation of

India into a supplier of foodstuffs and raw-materials to the metropolis, a market for

metropolitan manufacturers and a field for investment of British capital.

• These early nationalist analysts organised intellectual agitations and advocated a

complete severance of India’s economic subservience to Britain and the development

of an independent economy based on modern industries.

65. Consider the following statements regarding Press reforms

1. Indian Press Act, 1910 reversed the worst features of the Vernacular Press Act.

2. Indian Press Act, 1910 is popularly known as Metcalfe Act.

Which of the statements given above is/are correct?

(a) 1 only

(b) 2 only

(c) Both 1 and 2

(d) Neither 1 nor 2

Solution: D

• In 1883, Surendranath Banerjea became the first Indian journalist to be imprisoned.

In an angry editorial in The Bengalee Banerjea had criticised a judge of Calcutta High

Court for being insensitive to the religious sentiments of Bengalis in one of his

judgements.

• Bal Gangadhar Tilak is most frequently associated with the nationalist fight for the

freedom of press.

• Indian Press Act, 1910: This Act revived the worst features of the VPA—local

government was empowered to demand a security at registration from the

printer/publisher and fortfeit/deregister if it was an offending newspaper, and the

printer of a newspaper was required to submit two copies of each issue to local

government free of charge.

Press Act of 1835 or Metcalfe Act

• Metcalfe (governor-general—1835-36) repealed the obnoxious 1823 ordinance and

earned the epithet, “liberator of the Indian press”. The new Press Act (1835) required

a printer/publisher to give a precise account of premises of a publication and cease

functioning, if required by a similar declaration.

Page 61: SIMPLYFYING IAS EXAM PREPARATION...• He also founded the organisation Shri Mathura Vrindavan Hasanand Gochar Bhoomi in Vrindavan. 5. Periyar E. V. Ramasamy was associated with which

INSTA 75 Days REVISION PLAN for Prelims 2020 - InstaTests

www.insightsonindia.com 59 Insights IAS

66. Consider the following statements regarding Wood’s Despatch

1. It is known as Magna Carta of English education in India

2. It advocated downward filtration theory.

3. It recommended English as the medium of instruction for higher studies and at

school level.

Which of the statements given above is/are correct?

(a) 1 only

(b) 2 and 3 only

(c) 1 and 2 only

(d) 1, 2 and 3

Solution: A

• Lord Macaulay’s Minute (1835) advocated ‘downward filtration theory’.

Wood’s Despatch (1854)

• In 1854, Charles Wood prepared a despatch on an educational system for India.

Considered the “Magna Carta of English Education in India”, this document was the

first comprehensive plan for the spread of education in India.

• It asked the government of India to assume responsibility for education of the masses,

thus repudiating the ‘downward filtration theory’, at least on paper.

• It systematised the hierarchy from vernacular primary schools in villages at bottom,

followed by Anglo-Vernacular High Schools and an affiliated college at the district

level, and affiliating universities in the presidency towns of Calcutta, Bombay and

Madras.

• It recommended English as the medium of instruction for higher studies and

vernaculars at school level.

• It laid stress on female and vocational education, and on teachers’ training.

• It laid down that the education imparted in government institutions should be

secular.

• It recommended a system of grants-in-aid to encourage private enterprise.

67. Consider the following statements regarding Saddler University Commission

1. It held the view that, for the improvement of university education, improvement

of secondary education was a necessary pre-condition.

2. Based on its recommendations, the Indian Universities Act was passed in 1904.

Which of the statements given above is/are correct?

Page 62: SIMPLYFYING IAS EXAM PREPARATION...• He also founded the organisation Shri Mathura Vrindavan Hasanand Gochar Bhoomi in Vrindavan. 5. Periyar E. V. Ramasamy was associated with which

INSTA 75 Days REVISION PLAN for Prelims 2020 - InstaTests

www.insightsonindia.com 60 Insights IAS

(a) 1 only

(b) 2 only

(c) Both 1 and 2

(d) Neither 1 nor 2

Solution: A

Saddler University Commission (1917-19)

• The commission was set up to study and report on problems of Calcutta University but

its recommendations were applicable more or less to other universities also. It

reviewed the entire field from school education to university education. It held the

view that, for the improvement of university education, improvement of secondary

education was a necessary pre-condition.

• In 1902, Raleigh Commission was set up to go into conditions and prospects of

universities in India and to suggest measures for improvement in their constitution

and working. The commission precluded from reporting on primary or secondary

education. Based on its recommendations, the Indian Universities Act was passed in

1904.

68. Consider the following statements regarding Central Adoption Resource Authority

(CARA)

1. It is a statutory body established under Juvenile Justice Act, 2015.

2. It functions as a nodal body for adoption of Indian children and is mandated to

monitor and regulate in-country and inter-country adoption.

Which of the statements given above is/are not correct?

(a) 1 only

(b) 2 only

(c) Both 1 and 2

(d) Neither 1 nor 2

Solution: D

Central Adoption Resource Authority (CARA) is a statutory body of Ministry of Women &

Child Development established under Juvenile Justice Act, 2015.

Page 63: SIMPLYFYING IAS EXAM PREPARATION...• He also founded the organisation Shri Mathura Vrindavan Hasanand Gochar Bhoomi in Vrindavan. 5. Periyar E. V. Ramasamy was associated with which

INSTA 75 Days REVISION PLAN for Prelims 2020 - InstaTests

www.insightsonindia.com 61 Insights IAS

• CARA functions as a nodal body for adoption of Indian children and is mandated to

monitor and regulate in-country and inter-country adoption.

• It is designated as the Central Authority to deal with inter-country adoptions in

accordance with the provisions of the Hague Conventions on Inter-Country

Adoptions, 1993, ratified by Government of India in 2003.

• CARA primarily deals with adoption of orphan, abandoned and surrendered children

through its associated /recognised adoption agencies

http://cara.nic.in/about/about_cara.html

69. Consider the following statements regarding Islands Development agency

1. It was constituted in 2017 for preserving the natural eco-system and addressing

the security concerns.

2. It is a statutory body constituted under Island Development Act.

Which of the statements given above is/are correct?

(a) 1 only

(b) 2 only

(c) Both 1 and 2

(d) Neither 1 nor 2

Solution: A

• The Union Government constituted the Island Development Agency on 1st June, 2017

for the development of islands.

• It aims to preserve the natural eco-system and addressing the security concerns.

• It is non-statutory body. It is headed by Home Minister.

http://pib.nic.in/newsite/PrintRelease.aspx?relid=168895

70. Consider the following statements

1. Bengal Gazette was started by James Augustus Hicky.

2. India Gazette, First Bengali newspaper was started by Harishchandra Ray.

Which of the statements given above is/are correct?

(a) 1 only

(b) 2 only

(c) Both 1 and 2

Page 64: SIMPLYFYING IAS EXAM PREPARATION...• He also founded the organisation Shri Mathura Vrindavan Hasanand Gochar Bhoomi in Vrindavan. 5. Periyar E. V. Ramasamy was associated with which

INSTA 75 Days REVISION PLAN for Prelims 2020 - InstaTests

www.insightsonindia.com 62 Insights IAS

(d) Neither 1 nor 2

Solution: A

• Bengal Gazette was started by James Augustus Hicky in 1780, Calcutta.

• India Gazette was started in 1787. Henry Louis Vivian Derozio was associated with it.

• The India Gazette; or, Calcutta Public Advertiser was an English language weekly

newspaper published in Kolkata (then Calcutta), the capital of British India. It was the

second newspaper printed in India. Founded by Bernard Messink and Peter Reed, two

East India Company employees, the paper was a strong supporter of the

administration of the Governor General Warren Hastings, and a rival to India’s first

newspaper Hicky’s Bengal Gazette.

71. Match the following newspaper/journal with their founders

1. Sambad Kaumudi Girish Chandra Ghosh 2. Indian Mirror Devendra Nath Tagore 3. Bande mataram Madam Bikaji Kama 4. Indian Socialist Shyamji Krishna Varma

Which of the pairs given above is/are matched correctly?

(a) 1, 2 and 3 only

(b) 2, 3 and 4 only

(c) 1, 3 and 4 only

(d) 1, 2, 3 and 4

Solution: B

• Sambad Kaumudi (1821) (Bengali)- Ram Mohan Roy

• Mirat-Ul-Akbar (1822)1st Journal in Persian)-Ram Mohan Roy

• Banga duta-1822-A weekly in 4 languages.(English,Bengali, Persian,Hindi)Rajaram

Mohan Roy, Dwaraka nath Tagore and others.

• Jam-i-jahan numah(1822)1st news paper in Urdu- An English Firm

• Indian Mirror-Early 1862-1st English Daily-Devendra Nath Tagore

• Som Prakash-1858-Ishwar Chandra Vidyasagar

• Bengalee-1862-Girish Chandra Gosh-Taken over by S.N.Benerjea in 1879.

Page 65: SIMPLYFYING IAS EXAM PREPARATION...• He also founded the organisation Shri Mathura Vrindavan Hasanand Gochar Bhoomi in Vrindavan. 5. Periyar E. V. Ramasamy was associated with which

INSTA 75 Days REVISION PLAN for Prelims 2020 - InstaTests

www.insightsonindia.com 63 Insights IAS

• Madras mail-1868-1st evening News paper.

• Amrita Bazar Patrika-1868-(1st in Bengali later in English)Sisir Kumar Ghosh and

Motilal Ghosh

• Banga darshana-1873-Bankim chandra chaterjea.

• Indina statesman-1875-(Later THE STATESMAN)Robert Knight.

• The Hindu-1878-Veera Raghavachari, Subbha rao pandit and G.S.Aiyar

• Tribune-1881-Dayal singh Majeetia

• Kesari-1881-(Marati) B.G.Tilak

• Maharatta-1881-(English)B.G.Tilak

• Swadesha mitram—————-G.S.Aiyar

• Paridasak-1886-Bipan chandrapal(Publisher)

• New India (Weekly)-Bipin Chandra Pal

• New India (Daily)-Annie Besant

• Commonweal-Annie Besant

• Yugantar-1906-Bhupendranath Datta and Barinder Kumar Ghosh

• Sandhya-1906-Brahma Bandhav Upadhyaya

• Indian Socialist-(from London)Shyamji Krishna Varma.

• Bande mataram—Paris—Madam Bikaji Kama.

72. Which of the following events was/were took place under The Lord Lytton?

1. Vernacular Press Act

2. Ilbert Bill

3. 1st Delhi Durbar

Select the correct answer using the code given below

(a) 1 and 2 only

(b) 2 and 3 only

(c) 1 and 3 only

(d) 1, 2 and 3

Solution: C

The Lord Lytton (1831–1891)

12 April 1876

8 June 1880

• Vernacular Press Act, 1878

• Second Anglo-Afghan War, (1878–80)

• 1st Delhi Durbar out of 3

• Queen Victoria assuming the title of ‘Empress of India’

Page 66: SIMPLYFYING IAS EXAM PREPARATION...• He also founded the organisation Shri Mathura Vrindavan Hasanand Gochar Bhoomi in Vrindavan. 5. Periyar E. V. Ramasamy was associated with which

INSTA 75 Days REVISION PLAN for Prelims 2020 - InstaTests

www.insightsonindia.com 64 Insights IAS

The Marquess of Ripon (1827–1909)

8 June 1880

13 December 1884

• First Factory Act (1881)

• Negotiable Instruments Act (1881)

• Repeal of the Vernacular Press Act (1882)

• Ilbert Bill (1883)

• Government resolution on local self-government (1882)

• Appointment of Education Commission under Sir William Hunter

• First complete Census

73. Digital Payment Index has been released by

(a) NITI Aayog

(b) National Payments Corporation of India

(c) Reserve Bank of India

(d) Ministry of Electronics and Information Technology

Solution: C

• The Reserve Bank of India (RBI) will issue periodic scores to assess the performance of

the digital payments industry on key operating parameters such as rural penetration

and innovation in existing modes and channels.

• RBI released Digital Payments Index (DPI), the ‘composite’ scoring system will allow

both consumers and stakeholders to better gauge local area developments in

infrastructure, access, demographic and acceptance related growth with reference

to broader domestic and global standards in digital payments.

74. Unani system of medicine was introduced to India during the rule of

(a) Delhi Sultanate

(b) Mughals

(c) Mauryan Kingdom

(d) None of the above

Solution: A

Page 67: SIMPLYFYING IAS EXAM PREPARATION...• He also founded the organisation Shri Mathura Vrindavan Hasanand Gochar Bhoomi in Vrindavan. 5. Periyar E. V. Ramasamy was associated with which

INSTA 75 Days REVISION PLAN for Prelims 2020 - InstaTests

www.insightsonindia.com 65 Insights IAS

• The term Yūnānī means, as the Perso-Arabic system of medicine was based on the

teachings of the Greek physicians Hippocrates and Galen.

• The term Unani (Greek) means Perso-Arabic system of medicine. It was introduced to

India in 13th century with the establishment of Delhi Sultanate.

• Unani system of medicine is the term for Perso-Arabic traditional medicine as

practiced in Mughal India and in Muslim culture in South Asia and modern day Central

Asia.

75. Which of the following countries shares border with Baltic Sea?

1. Russia

2. Sweden

3. Belarus

4. Norway

Select the correct answer using the code given below

(a) 2 and 4 only

(b) 1 and 2 only

(c) 2, 3 and 4 only

(d) 1 and 3 only

Solution: B

Page 68: SIMPLYFYING IAS EXAM PREPARATION...• He also founded the organisation Shri Mathura Vrindavan Hasanand Gochar Bhoomi in Vrindavan. 5. Periyar E. V. Ramasamy was associated with which

INSTA 75 Days REVISION PLAN for Prelims 2020 - InstaTests

www.insightsonindia.com 66 Insights IAS

DAY – 36 (InstaTest-36)

76. Consider the following statements regarding vacation bench of the Supreme Court

1. It is constituted by the President of India

2. President of India appoints judges for vacation bench.

3. Vacation bench is only allowed to take the cases of grave constitutional issues.

Which of the statements given above is/are correct?

(a) 1 and 3 only

(b) 2 and 3 only

(c) 1 and 2 only

(d) None

Solution: D

• A Vacation Bench of the Supreme Court is a special bench constituted by the Chief

Justice of India for hearing matters during summer vacation or winter holidays.

• The Chief Justice may appoint one or more Judges to hear all matters of an urgent

nature which under these rules may be heard by a Judge sitting singly.

• Litigants can approach the Supreme Court and, if the court decides that the plea is an

‘urgent matter’, the Vacation Bench hears the case on its merits.

• During vacations the court generally admits writs related to habeas corpus,

certiorari, prohibition and quo warranto matters for enforcement of any

fundamental right.

77. Which of the following events took place under the Governor General Lord Curzon

1. Partition of Bengal

2. Surat Split of Congress

3. Establishment of Muslim League

Select the correct answer using the code given below

(a) 1 only

(b) 2 and 3 only

(c) 1 and 3 only

(d) 1, 2 and 3

Page 69: SIMPLYFYING IAS EXAM PREPARATION...• He also founded the organisation Shri Mathura Vrindavan Hasanand Gochar Bhoomi in Vrindavan. 5. Periyar E. V. Ramasamy was associated with which

INSTA 75 Days REVISION PLAN for Prelims 2020 - InstaTests

www.insightsonindia.com 67 Insights IAS

Solution: A

The Lord Curzon of Kedleston (1859–1925)

• Partition of Bengal (1905)

• Official Secrets Act 1904 to curb free press

• 2st Delhi Durbar out of 3 (1903)

• Appointment of Police Commission under Sir Andrew Frazer

• Appointment of Raleigh University Commission (1902)

• Passing of Indian Universities Act 1904

• 2nd Swadeshi Movement (1905-1911) against Partition of Bengal by Lal-Bal-Pal-Aurbindo Ghosh)

• Establishment of Archaeological Survey of India

• Benaras Hindu Girl’s School was established by Annie Besant in 1904

• (He said, “India is the pivot of our Empire…. If the Empire loses any other part of its Dominion we can survive, but if we lose India, the sun of our Empire will have set.”

The Earl of Minto (1845–1914)

• Morley-Minto Reforms 1909 or The Indian Councils Act 1909

• Split in Congress in 1907

• Seditious meetings (prohibition) Act 1907 to curb extremist movement

• Establishment of Muslim League by Aga Khan III (1906)

• Indian Press Act, 1910

• Jamsetji Tata established TISCO in 1907

78. Consider the following statements regarding regional kingdoms

1. The founder of the independent principality of Awadh was Saadat Khan

2. Murshid Kuli Khan was the founder of the independent state of Bengal.

Which of the statements given above is/are correct?

(a) 1 only

(b) 2 only

(c) Both 1 and 2

(d) Neither 1 nor 2

Solution: C

Page 70: SIMPLYFYING IAS EXAM PREPARATION...• He also founded the organisation Shri Mathura Vrindavan Hasanand Gochar Bhoomi in Vrindavan. 5. Periyar E. V. Ramasamy was associated with which

INSTA 75 Days REVISION PLAN for Prelims 2020 - InstaTests

www.insightsonindia.com 68 Insights IAS

Awadh

• The founder of the independent principality of Awadh was Saadat Khan, popularly

known as Burhan-ul-Mulk. Saadat Khan was a Shia. He had joined in a conspiracy

against the Sayyid brothers, which resulted in his being given an increased mansab.

• Later, driven out of the court, he was prompted to found a new independent state.

Saadat Khan committed suicide due to pressure from Nadir Shah who was demanding

a huge booty from him. He was succeeded by Safdar Jang as the Nawab of Awadh.

Bengal

• Murshid Kuli Khan was the founder of the independent state of Bengal. He was a

capable ruler and made Bengal a prosperous state. He was succeeded in 1727 by his

son Shujaud-din. His successor, Sarfaraz Khan, was killed in 1740 by Alivardi Khan, the

deputy governor of Bihar at Gheria, who assumed power and made himself

independent of the Mughal emperor by giving yearly tribute.

79. Consider the following statements regarding National Consumer Disputes Redressal

Commission (NCDRC)

1. It is a quasi-judicial commission

2. There is no appellate mechanism against the decision of NCDRC

3. It is headed by a sitting or retired judge of the Supreme Court of India.

Which of the statements given above is/are correct?

(a) 2 only

(b) 1 only

(c) 1 and 3 only

(d) 1, 2 and 3

Solution: C

The National Consumer Disputes Redressal Commission (NCDRC) is a quasi-judicial

commission in India set up in 1988 under the Consumer Protection Act of 1986.

• The commission is headed by a sitting or retired judge of the Supreme Court of India.

• According to Section 21 of Consumer Protection Act, 1986, The National Consumer

shall have jurisdiction to entertain a complaint valued more than one crore

• He/she also have Appellate and Revisional jurisdiction from the orders of State

Commissions or the District fora as the case may be.

Page 71: SIMPLYFYING IAS EXAM PREPARATION...• He also founded the organisation Shri Mathura Vrindavan Hasanand Gochar Bhoomi in Vrindavan. 5. Periyar E. V. Ramasamy was associated with which

INSTA 75 Days REVISION PLAN for Prelims 2020 - InstaTests

www.insightsonindia.com 69 Insights IAS

• Section 23 of Consumer Protection Act, 1986, provides that any person aggrieved by

an order of NCDRC, may prefer an Appeal against such order to Supreme Court of India

within a period of 30 days.

80. Chatuspathis or Tols were referred to

(a) Village accountants

(b) Nomads and herders.

(c) Head of trade associations.

(d) Centers of higher education

Solution: D

• Chatuspathis or Tols, as they were called in Bihar and Bengal, were the centres of

higher education. Some of the famous centres for Sanskrit education were Kasi

(Varanasi), Tirhut (Mithila), Nadia and Utkala. Madrasahs were the institutions of

higher learning for Persian and Arabic, Persian being the court language and learnt by

the Muslims as well as the Hindus. Azimabad (Patna) was a famous centre for Persian

education. People interested in the study of the Quran and Muslim theology had to

acquire proficiency in Arabic.

81. Consider the following statements regarding Dual Government in Bengal

1. Warren Hasting introduced the dual system of government.

2. Diwani and nizamat came under the control of the Company.

3. The nawab was responsible for maintaining peace and order, but he was

depended on Company

Which of the statements given above is/are correct?

(a) 2 only

(b) 2 and 3 only

(c) 1 and 3 only

(d) 1, 2 and 3

Solution: B

Page 72: SIMPLYFYING IAS EXAM PREPARATION...• He also founded the organisation Shri Mathura Vrindavan Hasanand Gochar Bhoomi in Vrindavan. 5. Periyar E. V. Ramasamy was associated with which

INSTA 75 Days REVISION PLAN for Prelims 2020 - InstaTests

www.insightsonindia.com 70 Insights IAS

Dual Government in Bengal (1765-72)

• After the battle of Buxar, the East India Company became the real masters of Bengal.

Robert Clive introduced the dual system of government, i.e., the rule of the two—

the Company and the Nawab—in Bengal in which both the diwani, i.e., collecting

revenues, and nizamat, i.e., police and judicial functions, came under the control of

the Company. The Company exercised diwani rights as the diwan and the nizamat

rights through its right to nominate the deputy subahdar. The Company acquired the

diwani functions from the emperor and nizamat functions from the subahdar of

Bengal.

• The system held a great advantage for the Company. It left the appearance of

authority to the puppet Indian ruler, while keeping the sovereign power in the hands

of the Company. The nawab was responsible for maintaining peace and order, but he

depended both for funds and forces upon the Company because the latter controlled

the army and revenues.

82. Consider the following statements regarding Antrix

1. It is the commercial arm of Indian Space Research Organisation

2. It provides launch services for customer satellites

Which of the statements given above is/are correct?

(a) 1 only

(b) 2 only

(c) Both 1 and 2

(d) Neither 1 nor 2

Solution: C

Antrix is the commercial arm of Indian Space Research Organisation (ISRO). Antrix

Corporation Limited (Antrix) was incorporated on 1992 under the Companies Act, 1956, to

market the products and services of the Indian Space Research Organisation (ISRO).

• It is a wholly Government of India owned Company under the administrative control

of Department of Space (DOS).

• In the year 2008, the Company was awarded ‘MINIRATNA’ status.

Current business activities of Antrix

• Provisioning of communication satellite transponders to various users

• Providing launch services for customer satellites

• Marketing of data from Indian and foreign remote sensing satellites

• Building and marketing of satellites as well as satellite sub-systems

Page 73: SIMPLYFYING IAS EXAM PREPARATION...• He also founded the organisation Shri Mathura Vrindavan Hasanand Gochar Bhoomi in Vrindavan. 5. Periyar E. V. Ramasamy was associated with which

INSTA 75 Days REVISION PLAN for Prelims 2020 - InstaTests

www.insightsonindia.com 71 Insights IAS

• Establishing ground infrastructure for space applications

• Mission support services for satellites.

http://www.antrix.co.in/about-us

83. Consider the following statements regarding Dadabhai Naoroji

1. He served as the first Indian member of the British parliament.

2. He was Congress president thrice.

3. He became a member of the royal commission on Indian expenditure.

Which of the statements given above is/are correct?

(a) 1 and 2 only

(b) 2 and 3 only

(c) 1 and 3 only

(d) 1, 2 and 3

Solution: D

Dadabhai Naoroji

• Dadabhai Naoroji, the “Grand Old Man of India”, who was among the first leaders

who stirred national consciousness in the country.

• Born in 1825 at Navsari, in present-day Gujarat.

Key contributions:

• He was closely involved with the Indian National Congress in its early phase.

• He served as the first Indian member of the British parliament.

• His first agitation, in 1859, concerned recruitment to the Indian Civil Service.

• In 1865 and 1866, Naoroji helped found the London Indian Society and the East India

Association. The two organisations sought to bring nationalist Indians and

sympathetic Britons on one platform.

• As the secretary of the East India Association, Naoroji travelled in India to gather funds

and raise national awareness.

• In 1885, Naoroji became a vice-president of the Bombay Presidency Association, was

nominated to the Bombay legislative council by Governor Lord Reay, and helped form

the Indian National Congress.

• He was Congress president thrice, in 1886, 1893, and 1906.

• In 1893, he helped form an Indian parliamentary committee to attend to Indian

interests.

• In 1895, he became a member of the royal commission on Indian expenditure.

Page 74: SIMPLYFYING IAS EXAM PREPARATION...• He also founded the organisation Shri Mathura Vrindavan Hasanand Gochar Bhoomi in Vrindavan. 5. Periyar E. V. Ramasamy was associated with which

INSTA 75 Days REVISION PLAN for Prelims 2020 - InstaTests

www.insightsonindia.com 72 Insights IAS

• Dadabhai Naoroji was among the key proponents of the ‘Drain Theory’, disseminating

it in his 1901 book ‘Poverty and Un-British Rule in India’.

What is Drain Theory?

• Imperial Britain was draining away India’s wealth to itself through exploitative

economic policies, including India’s rule by foreigners; the heavy financial burden of

the British civil and military apparatus in India; the exploitation of the country due to

free trade; non-Indians taking away the money that they earned in India; and the

interest that India paid on its public debt held in Britain.

84. Consider the following statements regarding Wahabi Movement

1. The Wahabi Movement was essentially an Islamic revivalist movement founded

by Syed Ahmed

2. Syed Ahmed condemned the western influence on Islam and advocated a return

to pure Islam

3. The Wahabis played an important role in spreading Pro-British sentiments.

Which of the statements given above is/are correct?

(a) 1 and 2 only

(b) 2 only

(c) 1 and 3 only

(d) 1, 2 and 3

Solution: A

Wahabi Movement

• The Wahabi Movement was essentially an Islamic revivalist movement founded by

Syed Ahmed of Rai Bareilly who was inspired by the teachings of Abdul Wahab (1703-

87) of Saudi Arabia and Shah Waliullah of Delhi. Syed Ahmed condemned the western

influence on Islam and advocated a return to pure Islam and society as it was in the

Arabia of the Prophet’s time.

• Syed Ahmed was acclaimed as the desired leader (Imam). A countrywide

organisation with an elaborate secret code for its working under spiritual vice-regents

(Khalifas) was set up, and Sithana in the north-western tribal belt was chosen as a

base for operations. In India, its important centre was at Patna though it had its

missions in Hyderabad, Madras, Bengal, United Provinces and Bombay. Since Dar-ul-

Harb (the land of kafirs) was to be converted into Dar-ul-Islam (the land of Islam), a

jihad was declared against the Sikh kingdom of Punjab. After the defeat of the Sikh

Page 75: SIMPLYFYING IAS EXAM PREPARATION...• He also founded the organisation Shri Mathura Vrindavan Hasanand Gochar Bhoomi in Vrindavan. 5. Periyar E. V. Ramasamy was associated with which

INSTA 75 Days REVISION PLAN for Prelims 2020 - InstaTests

www.insightsonindia.com 73 Insights IAS

ruler and incorporation of Punjab into the East India Company’s dominion in 1849, the

English dominion in India became the sole target of the Wahabis’ attacks.

• The Wahabis played an important role in spreading anti-British sentiments. A series

of military operations by the British in the 1860s on the Wahabi base in Sithana and

various court cases of sedition on the Wahabis weakened the Wahabi resistance,

although sporadic encounters with the authorities continued into the 1880s and

1890s.

85. Which of the following rights is/are entitled to the farmers by Protection of Plant Variety

and Farmers Right Act, 2001

1. Farmers are entitled to save, use, sow, re-sow, exchange, share or sell their farm

produce, including seed of protected varieties

2. Plant breeders and legal entities including farmers who provide Plant Genetic

Resources (PGR) to breeders for developing new varieties shall receive a fair share

of benefit from the commercial gains

3. Farmers have the right to access seed of registered varieties at a reasonable and

remunerative price.

Select the correct answer using the code given below

(a) 2 and 3 only

(b) 2 only

(c) 1 and 3 only

(d) 1, 2 and 3

Solution: D

About Protection of Plant Variety and Farmers Right Act, 2001

• The Protection of Plant Varieties and Farmers’ Rights Act (PPV&FR Act) seeks to

address the rights of plant breeders and farmers on an equal footing. It affirms the

necessity of recognizing and protecting the rights of farmers with respect to the

contribution they make in conserving, improving and making Plant Genetic Resources

(PGR) available for the development of new plant varieties.

Right 1: Access to seed [Section 39(1)(iv)]

• Farmers are entitled to save, use, sow, re-sow, exchange, share or sell their farm

produce, including seed of protected varieties, in the same manner as they were

entitles to before the coming into force to the PPV&FR Act. However, farmers are not

entitled to sell branded seed of a variety protected under this Act. Farmers can use

farm saved seed from a crop cultivated in their own.

Page 76: SIMPLYFYING IAS EXAM PREPARATION...• He also founded the organisation Shri Mathura Vrindavan Hasanand Gochar Bhoomi in Vrindavan. 5. Periyar E. V. Ramasamy was associated with which

INSTA 75 Days REVISION PLAN for Prelims 2020 - InstaTests

www.insightsonindia.com 74 Insights IAS

Right 2: Benefit sharing [Section 26]

• Plant breeders and legal entities including farmers who provide Plant Genetic

Resources (PGR) to breeders for developing new varieties shall receive a fair share of

benefit from the commercial gains of the registered varieties. Out of all the national

plant variety protection laws enacted since 2001, the PPV&FR Act is the first that

integrates a provision for access and benefit-sharing (ABS) along with Plant Breeder’s

Rights (PBRs). Accession of the genetic resource used in breeding is permitted under

the Biological Diversity Act, 2002. However, the PPV&FR Act requires a breeder to

make a sworn declaration on the geographical origin of the genetic resources used in

the pedigree of the new variety, and its accession.

Right 3: Compensation [Section 39(2)]

• Registered seed must be sold with the full disclosure of their agronomic performance

under recommended management conditions. When such seed is sold to farmers

but fails to provide the expected performance under recommended management

conditions, the farmer is eligible to claim compensation from the breeder through

the intervention of the PPV&FR Authority.

https://vikaspedia.in/agriculture/policies-and-schemes/crops-related/protection-of-plant-

varieties-and-rights-of-farmers/farmers2019-rights-in-the-ppv-fr-act-2001

86. Consider the following statements regarding women organization

1. Sarla Devi Chaudhurani convened the first meeting of the Bharat Stree

Mahamandal.

2. Mehribai Tata founded the Ladies Social Conference.

3. Pandita Ramabai Saraswati founded the Arya Mahila Samaj.

Which of the statements given above is/are correct?

(a) 1 only

(b) 2 and 3 only

(c) 1 and 3 only

(d) 1, 2 and 3

Solution: C

• In 1910, Sarla Devi Chaudhurani convened the first meeting of the Bharat Stree

Mahamandal in Allahabad.

• Considered as the first major Indian women’s organisation set up by a woman, its

objectives included promotion of education for women, abolition of the purdah

Page 77: SIMPLYFYING IAS EXAM PREPARATION...• He also founded the organisation Shri Mathura Vrindavan Hasanand Gochar Bhoomi in Vrindavan. 5. Periyar E. V. Ramasamy was associated with which

INSTA 75 Days REVISION PLAN for Prelims 2020 - InstaTests

www.insightsonindia.com 75 Insights IAS

system and improvement in the socio-economic and political status of woman all

over India. Sarla Devi believed that the man working for women’s upliftment lived

‘under the shade of Manu’.

• Ramabai Ranade founded the Ladies Social Conference (Bharat Mahila Parishad),

under the parent organization National Social Conference, in 1904 in Bombay.

• Pandita Ramabai Saraswati founded the Arya Mahila Samaj to serve the cause of

women. She pleaded for improvement in the educational syllabus of Indian women

before the English Education Commission which was referred to Queen Victoria. This

resulted in medical education for women which started in Lady Dufferin College.

Later Ramabai Ranade established a branch of Arya Mahila Samaj in Bombay.

• In 1925, the National Council of Women in India, a national branch of the

International Council of Women, was formed. Mehribai Tata played a vital role in its

formation and advancement.

87. Consider the following pairs of political associations and their founders

Political Associations Founders 1. The Madras Mahajan

Sabha : M. Viraraghavachari,

B. Subramaniya Aiyer and P. Anandacharlu

2. The Bombay Presidency Association

: Mahadeo Govind Ranade

3. The Poona Sarvajanik Sabha

: Badruddin Tyabji, Pherozshah Mehta and K.T. Telang

Which of the pairs given above is/are correctly matched?

(a) 1 only

(b) 2 and 3 only

(c) 1 and 3 only

(d) 1, 2 and 3

Solution: A

Political Associations in Bombay

• The Poona Sarvajanik Sabha was founded in 1867 by Mahadeo Govind Ranade and

others, with the object of serving as a bridge between the government and the people.

• The Bombay Presidency Association was started by Badruddin Tyabji, Pherozshah

Mehta and K.T. Telang in 1885.

Page 78: SIMPLYFYING IAS EXAM PREPARATION...• He also founded the organisation Shri Mathura Vrindavan Hasanand Gochar Bhoomi in Vrindavan. 5. Periyar E. V. Ramasamy was associated with which

INSTA 75 Days REVISION PLAN for Prelims 2020 - InstaTests

www.insightsonindia.com 76 Insights IAS

Political Associations in Madras

• The Madras Mahajan Sabha was founded in 1884 by M. Viraraghavachari, B.

Subramaniya Aiyer and P. Anandacharlu.

88. B N Srikrishna committee, sometime seen in the news, is related to

(a) Resolving Kashmir Issue

(b) Universal Basic Income

(c) Data Protection

(d) None of the above

Solution: C

• Cleared by the Cabinet, the Personal Data Protection Bill is due to be placed in

Parliament.

• The Bill has three key aspects that were not previously included in a draft version,

prepared by a committee headed by retired Justice B N Srikrishna.

How does the bill seek to regulate data?

The bill constitutes 3 personal information types:

1. Critical

2. Sensitive

3. General

1. Sensitive data constitutes or is related to passwords, financial data, health data, official

identifier, sexual orientation, religious or caste data, biometric data and genetic data. It may

be processed outside India with the explicit consent of the user.

2. Critical data will be characterised by the government every once in a while, and must be

stored and handled only in India.

3. General data: Any data that is non-critical and non-sensitive is categorised as general data

with no limitation on where it is stored or managed.

Other Key provisions:

• Data principal: As per the bill, it is the individual whose data is being stored and

processed.

• Exemptions: The government is qualified to order any data fiduciary to acquire

personal and non-personal/anonymised data for the sake of research and for national

security and criminal investigations.

Page 79: SIMPLYFYING IAS EXAM PREPARATION...• He also founded the organisation Shri Mathura Vrindavan Hasanand Gochar Bhoomi in Vrindavan. 5. Periyar E. V. Ramasamy was associated with which

INSTA 75 Days REVISION PLAN for Prelims 2020 - InstaTests

www.insightsonindia.com 77 Insights IAS

• Social media companies, which are deemed significant data fiduciaries based on

factors such as volume and sensitivity of data as well as their turnover, should develop

their own user verification mechanism.

• An independent regulator Data Protection Agency (DPA) will oversee assessments

and audits and definition making.

• Each company will have a Data Protection Officer (DPO) who will liaison with the DPA

for auditing, grievance redressal, recording maintenance and more.

• The bill also grants individuals the right to data portability, and the ability to access

and transfer one’s own data.

• The right to be forgotten: this right allows an individual to remove consent for data

collection and disclosure.

https://www.insightsonindia.com/2019/12/07/data-protection-bill/

89. Consider the following statements regarding Lucknow Session (1916)

1. It was presided over by Ambika Charan Majumdar.

2. Readmission of Extremists to Congress with the efforts of Annie Besant and Tilak.

3. Lucknow Pact between Congress and Muslim League, led to acceptance of

separate electorate.

Which of the statements given above is/are correct?

(a) 1 only

(b) 2 and 3 only

(c) 1 and 3 only

(d) 1, 2 and 3

Solution: D

Lucknow Session of the Indian National Congress (1916)

Readmission of Extremists to Congress

The Lucknow session of the Indian National Congress, presided over by a Moderate, Ambika

Charan Majumdar, finally readmitted the Extremists led by Tilak to the Congress fold.

Various factors facilitated this reunion:

1. Old controversies had become meaningless now.

2. Both the Moderates and the Extremists realized that the split had led to political

inactivity.

Page 80: SIMPLYFYING IAS EXAM PREPARATION...• He also founded the organisation Shri Mathura Vrindavan Hasanand Gochar Bhoomi in Vrindavan. 5. Periyar E. V. Ramasamy was associated with which

INSTA 75 Days REVISION PLAN for Prelims 2020 - InstaTests

www.insightsonindia.com 78 Insights IAS

3. Annie Besant and Tilak had made vigorous efforts for the reunion. To allay Moderate

suspicions, Tilak had declared that he supported a reform of administration and not an

overthrow of the government. He also denounced acts of violence.

4. The death of two Moderates, Gokhale and Pherozshah Mehta, who had led the Moderate

opposition to the Extremists, facilitated the reunion.

Lucknow Pact between Congress and Muslim League

• Another significant development to take place at Lucknow was the coming together

of the Muslim League and the Congress and the presentation of common demands by

them to the government. This happened at a time when the Muslim League, now

dominated by the younger militant nationalists, was coming closer to the Congress

objectives and turning increasingly anti-imperialist.

90. Consider the following statements regarding The Tolstoy Farm

1. The Tolstoy Farm was the first of its kind established by Gandhi.

2. It was to house the families of the satyagrahis.

Which of the statements given above is/are correct?

(a) 1 only

(b) 2 only

(c) Both 1 and 2

(d) Neither 1 nor 2

Solution: B

• As it became rather difficult to sustain the high pitch of the struggle, Gandhi decided

to devote all his attention to the struggle. The Tolstoy Farm was founded in 1910 and

named as such by Gandhi’s associate, Herman Kallenbach, after the Russian writer

and moralist, whom Gandhi admired and corresponded with. Besides being an

experiment in education, it was to house the families of the satyagrahis and to give

them a way to sustain themselves.

• The Tolstoy Farm was the second of its kind established by Gandhi. He had set up the

Phoenix Farm in 1904 in Natal, inspired by a reading of John Ruskin’s Unto This Last,

a critique of capitalism, and a work that extolled the virtues of the simple life of love,

labour, and the dignity of human beings. As at the Phoenix settlement, at Tolstoy Farm

too, manual work went hand-in-hand with instruction. Vocational training was

introduced to give “all-round development to the boys and girls”.

Page 81: SIMPLYFYING IAS EXAM PREPARATION...• He also founded the organisation Shri Mathura Vrindavan Hasanand Gochar Bhoomi in Vrindavan. 5. Periyar E. V. Ramasamy was associated with which

INSTA 75 Days REVISION PLAN for Prelims 2020 - InstaTests

www.insightsonindia.com 79 Insights IAS

91. Consider the following statements regarding Red Panda:

1. They are mainly found in the forests of India, Nepal, Bhutan and Myanmar and

China.

2. It has been listed as near threatened in IUCN’s Red list.

3. It is the state animal of Meghalaya.

Which of the statements given above is/are correct?

(a) 1 only

(b) 3 only

(c) 2 and 3 only

(d) 1 and 2 only

Solution: A

• The red panda is a small

arboreal mammal found in

the forests of India, Nepal,

Bhutan and the northern

mountains of Myanmar

and southern China.

• The red panda is slightly

larger than a domestic cat

with a bear-like body and

thick russet fur. The belly

and limbs are black, and

there are white markings

on the side of the head

and above its small eyes.

Red pandas are very

skillful and acrobatic

animals that

predominantly stay in

trees. Almost 50% of the

red panda’s habitat is in

the Eastern Himalayas. It is

the state animal of Sikkim.

• Clouded Leopard is the

state animal of

Meghalaya.

Page 82: SIMPLYFYING IAS EXAM PREPARATION...• He also founded the organisation Shri Mathura Vrindavan Hasanand Gochar Bhoomi in Vrindavan. 5. Periyar E. V. Ramasamy was associated with which

INSTA 75 Days REVISION PLAN for Prelims 2020 - InstaTests

www.insightsonindia.com 80 Insights IAS

92. Consider the following statements regarding Jallianwala Bagh Massacre

1. Following the incident, Rabindranath Tagore renounced his knighthood in protest.

2. Gandhi gave up the title of Kaiser-i-Hind following the incident.

3. Udham Singh assassinated Michael O’Dwyer, the Lieutenant-Governor who

presided over the brutal British suppression of the 1919 protests in Punjab.

Which of the statements given above is/are correct?

(a) 1 and 2 only

(b) 2 and 3 only

(c) 1 only

(d) 1, 2 and 3

Solution: D

Jallianwala Bagh Massacre (April 13, 1919)

• Amritsar was the worst affected by violence. In the beginning there was no violence

by the protestors. Indians shut down their shops and normal trade and the empty

streets showed the Indians’ displeasure at the British betrayal. On April 9, two

nationalist leaders, Saifuddin Kitchlew and Dr Satyapal, were arrested by the British

officials without any provocation except that they had addressed protest meetings,

and taken to some unknown destination. This caused resentment among the Indian

protestors who came out in thousands on April 10 to show their solidarity with their

leaders. Soon the protests turned violent because the police resorted to firing in which

some of the protestors were killed. Tension ran high.

• In the riot that followed, five Englishmen are reported to have been killed and

Marcella Sherwood, an English woman missionary going on a bicycle, was beaten up.

• On Baisakhi day, a large crowd of people mostly from neighbouring villages, unaware

of the prohibitory orders in the city, gathered in the Jallianwala Bagh, a popular place

for public events, to celebrate the Baisakhi festival. Local leaders had also called for a

protest meeting at the venue.

• The entire nation was stunned. Rabindranath Tagore renounced his knighthood in

protest. Gandhi gave up the title of Kaiser-i-Hind, bestowed by the British for his work

during the Boer War. Gandhi was overwhelmed by the atmosphere of total violence

and withdrew the movement on April 18, 1919.

• Udham Singh, who bore the name, Ram Mohammad Singh Azad, later assassinated

Michael O’Dwyer, the Lieutenant-Governor who presided over the brutal British

suppression of the 1919 protests in Punjab. Udham Singh was hanged in 1940 for his

deed. (His ashes were returned to India in 1974.)

Page 83: SIMPLYFYING IAS EXAM PREPARATION...• He also founded the organisation Shri Mathura Vrindavan Hasanand Gochar Bhoomi in Vrindavan. 5. Periyar E. V. Ramasamy was associated with which

INSTA 75 Days REVISION PLAN for Prelims 2020 - InstaTests

www.insightsonindia.com 81 Insights IAS

93. Consider the following statements regarding Hindustan Republican Association

1. The HRA was founded by Ramprasad Bismil, Jogesh Chandra Chatterjee and

Sachin Sanyal.

2. Under the sole leadership of Bhagat Singh, the name of HRA was changed to

Hindustan Socialist Republican Association (HSRA).

Which of the statements given above is/are correct?

(a) 1 only

(b) 2 only

(c) Both 1 and 2

(d) Neither 1 nor 2

Solution: A

• The revolutionary activity in this region was dominated by the Hindustan Republican

Association/Army or HRA (later renamed Hindustan Socialist Republican Association

or HSRA). The HRA was founded in October 1924 in Kanpur by Ramprasad Bismil,

Jogesh Chandra Chatterjee and Sachin Sanyal, with an aim to organise an armed

revolution to overthrow the colonial government and establish in its place the Federal

Republic of United States of India whose basic principle would be adult franchise.

The HSRA

• Determined to overcome the Kakori setback, the younger revolutionaries, inspired by

socialist ideas, set out to reorganize Hindustan Republic Association at a historic

meeting in the ruins of Ferozshah Kotla in Delhi (September 1928). Under the

leadership of Chandra Shekhar Azad, the name of HRA was changed to Hindustan

Socialist Republican Association (HSRA). The participants included Bhagat Singh,

Sukhdev, Bhagwaticharan Vohra from Punjab and Bejoy Kumar Sinha, Shiv Verma

and Jaidev Kapur from the United Provinces. The HSRA decided to work under a

collective leadership and adopted socialism as its official goal.

94. Consider the following statements regarding Katchatheevu islands:

1. It is located in the Palk Strait.

2. It was ceded to Sri Lanka through the Indo-Sri Lanka Peace Accord in 1989.

3. St. Antony’s shrine is located on the island.

Which of the statements given above is/are correct?

(a) 1 and 3 only

Page 84: SIMPLYFYING IAS EXAM PREPARATION...• He also founded the organisation Shri Mathura Vrindavan Hasanand Gochar Bhoomi in Vrindavan. 5. Periyar E. V. Ramasamy was associated with which

INSTA 75 Days REVISION PLAN for Prelims 2020 - InstaTests

www.insightsonindia.com 82 Insights IAS

(b) 2 only

(c) 1, 2 and 3

(d) 3 only

Solution: A

• Katchatheevu is an uninhabited islet in the Palk Strait. The island was earlier part of

the Ramnad Kingdom which existed in Madurai district of India. Later on with the

British rule on the Indian subcontinent was the island part of the Madras Presidency.

• In 1974, Katchatheevu was ceded to Sri Lanka through the Indo-Sri Lankan Maritime

agreement to settle the maritime boundary in the Palk Strait.

• In 1976 through another accord, India further gave up its fishing rights in the region.

• St. Antony’s shrine is the only structure on the island. It is a church named after St.

Antony, considered a patron-saint of seafarers by the Christians. It was built by a

prosperous Indian an Indian Catholic (Tamilian) fisherman Srinivasa Padaiyachi in the

early 20th century. The annual church festival runs for three days. The priests from

both India and Sri Lanka conduct the mass and car procession. Pilgrims from India are

ferried mostly Rameswaram. According to the agreement between the Indian and Sri

Lankan government, the citizens of India are not required to possess an Indian

passport or Sri Lankan visa for visiting Kachchatheevu.

Page 85: SIMPLYFYING IAS EXAM PREPARATION...• He also founded the organisation Shri Mathura Vrindavan Hasanand Gochar Bhoomi in Vrindavan. 5. Periyar E. V. Ramasamy was associated with which

INSTA 75 Days REVISION PLAN for Prelims 2020 - InstaTests

www.insightsonindia.com 83 Insights IAS

https://www.thehindu.com/news/cities/Madurai/allow-country-boats-to-katchatheevu-

church-festival/article30667291.ece

95. Consider the following statements regarding education reforms

1. The Calcutta Madrasah was established by Jonathan Duncan

2. The Sanskrit College was established by Warren Hastings

Which of the statements given above is/are correct?

(a) 1 only

(b) 2 only

(c) Both 1 and 2

(d) Neither 1 nor 2

Solution: D

• The Calcutta Madrasah was established by Warren Hastings in 1781 for the study of

Muslim law and related subjects.

• The Sanskrit College was established by Jonathan Duncan, the resident, at Benaras in

1791 for study of Hindu law and philosophy.

• Fort William College was set up by Wellesley in 1800 for training of civil servants of

the Company in languages and customs of Indians (closed in 1802).

• The Calcutta Madrasah and the Sanskrit College were designed to provide a regular

supply of qualified Indians to help the administration of law in the Company’s court,

and the knowledge of classical languages and vernaculars was useful in

correspondence with Indian states.

96. Consider the following statements regarding the Hydroxychloroquine (HCQ):

1. It is used to prevent and treat malaria in areas where malaria remains sensitive to

chloroquine.

2. It is listed on the World Health Organization’s List of Essential Medicines,

3. It is used in the treatment of rheumatoid arthritis.

Which of the statements given above is/are correct?

(a) 1 and 2 only

(b) 3 only

(c) 2 and 3 only

Page 86: SIMPLYFYING IAS EXAM PREPARATION...• He also founded the organisation Shri Mathura Vrindavan Hasanand Gochar Bhoomi in Vrindavan. 5. Periyar E. V. Ramasamy was associated with which

INSTA 75 Days REVISION PLAN for Prelims 2020 - InstaTests

www.insightsonindia.com 84 Insights IAS

(d) 1, 2 and 3

Solution: D

• Hydroxychloroquine (HCQ), sold under the brand name Plaquenil among others, is a

medication used to prevent and treat malaria in areas where malaria remains

sensitive to chloroquine. Other uses include treatment of rheumatoid arthritis, lupus,

and porphyria cutanea tarda. It is taken by mouth. It is also being studied as a

treatment for coronavirus disease 2019 (COVID-19).

https://m.economictimes.com/industry/healthcare/biotech/pharmaceuticals/india-will-

allow-some-exports-of-anti-malaria-drug-after-trump-appeal/articleshow/75022468.cms

97. Consider the following pairs of the British officers who resisted the 1857 revolt and

respective revolt centres:

Revolt Centre British Officer 1. Delhi : John Nicholson 2. Kanpur : Sir Colin Campbell 3. Jhansi : Sir Hugh Rose 4. Benaras : Colonel James Neill

Which of the pairs given above is/are correctly matched?

(a) 1, 2 and 3 only

(b) 2, 3 and 4 only

(c) 1, 3 and 4 only

(d) 1, 2, 3 and 4

Solution: D

• Indian Mutiny, also called Sepoy Mutiny or First War of Independence, widespread

but unsuccessful rebellion against British rule in India in 1857–59. Begun in Meerut by

Indian troops (sepoys) in the service of the British East India Company, it spread to

Delhi, Agra, Kanpur, and Lucknow. It is often called the First War of Indian

Independence and other similar names.

Page 87: SIMPLYFYING IAS EXAM PREPARATION...• He also founded the organisation Shri Mathura Vrindavan Hasanand Gochar Bhoomi in Vrindavan. 5. Periyar E. V. Ramasamy was associated with which

INSTA 75 Days REVISION PLAN for Prelims 2020 - InstaTests

www.insightsonindia.com 85 Insights IAS

The British Resistance to 1857 revolt

Revolt Centre and British officers who led the British troops are as follows:

1. Delhi – Lieutenant Willoughby, John Nicholson, Lieutenant Hudson

2. Kanpur – Sir Hugh Wheeler, Sir Colin Campbell

3. Lucknow – Henry Lawrence, Brigadier Inglis, Henry Havelock, James Outram, Sir Colin

Campbell

4. Jhansi – Sir Hugh Rose

5. Benaras – Colonel James Neill

98. Which of the following organizations releases the Gender Social Norms Index (GSNI)?

(a) World Bank

(b) World Economic Forum (WEF)

(c) The United Nations Educational, Scientific and Cultural Organization (UNESCO)

(d) The United Nations Development Programme (UNDP)

Solution: D

• UNDP releases the Gender Social Norms Index (GSNI). The Gender Social Norms

Index (GSNI) measures how social beliefs obstruct gender equality in areas like

politics, work, and education, and contains data from 75 countries, covering over 80

percent of the world’s population.

http://hdr.undp.org/en/gsni

99. Consider the following statements regarding Tebhaga Movement

1. Flood Commission had recommended one-half share to the bargardars and share-

croppers.

2. Muslims participated in large numbers in this movement.

Which of the statements given above is/are correct?

(a) 1 only

(b) 2 only

(c) Both 1 and 2

(d) Neither 1 nor 2

Page 88: SIMPLYFYING IAS EXAM PREPARATION...• He also founded the organisation Shri Mathura Vrindavan Hasanand Gochar Bhoomi in Vrindavan. 5. Periyar E. V. Ramasamy was associated with which

INSTA 75 Days REVISION PLAN for Prelims 2020 - InstaTests

www.insightsonindia.com 86 Insights IAS

Solution: B

Tebhaga Movement

• In September 1946, the Bengal Provincial Kisan Sabha gave a call to implement,

through mass struggle, the Flood Commission recommendations of tebhaga—two-

thirds’ share—to the bargardars, the share-croppers also known as bagchasi or

adhyar, instead of the one-half share. The bargardars worked on lands rented from

the jotedars. The communist cadres, including many urban student militias went to

the countryside to organise the bargardars. The central slogan was “nij khamare dhan

tolo”—i.e., sharecroppers taking the paddy to their own threshing floor and not to the

jotedar’s house, as before, so as to enforce tebhaga.

• The storm centre of the movement was north Bengal, principally among Rajbanshis—

a low caste of tribal origin.

• Muslims also participated in large numbers. The movement dissipated soon, because

of the League ministry’s sop of the Bargardari Bill, an intensified repression, the

popularization of the Hindu Mahasabha’s agitation for a separate Bengal and renewed

riots in Calcutta which ended the prospects of sympathetic support from the urban

sections.

100. Consider the following statements regarding Kurzarbeit scheme/policy:

1. It was rolled out by the Brazil government during 2008-09 global economic crisis.

2. It aims to address workers who are impacted by loss of income due to shortened

work hours during economic crisis.

Which of the statements given above is/are correct?

(a) 1 only

(b) 2 only

(c) Both 1 and 2

(d) Neither 1 nor 2

Solution: B

• Amid the all-round disruption caused to the economy by the novel coronavirus

outbreak, a concern across the world is the possibility of loss of jobs. Various

governments have unveiled various measures to address such concerns, and one of

the most talked about is Kurzarbeit, Germany’s existing scheme that provides for

Page 89: SIMPLYFYING IAS EXAM PREPARATION...• He also founded the organisation Shri Mathura Vrindavan Hasanand Gochar Bhoomi in Vrindavan. 5. Periyar E. V. Ramasamy was associated with which

INSTA 75 Days REVISION PLAN for Prelims 2020 - InstaTests

www.insightsonindia.com 87 Insights IAS

partial compensation for a worker’s earnings in such situations, and now modified to

account for the current crisis.

• The policy was rolled out during the 2008 economic crisis while its origins date back

as far as the early 20th century, before and after World War I.

• When companies face a loss of earnings due to unforeseen economic situations, they

often need to cut back on their working hours or send some of their employees home.

The Kurzarbeit scheme aims to address workers who are impacted by loss of income

due to shortened work hours during such times. They can apply for short-term work

benefits under the scheme, with the government stepping in to pay employees a part

of their lost income. This helps the companies retain their employees instead of

laying them off, and allows the latter to sustain themselves for a period of up to 12

months.

https://indianexpress.com/article/explained/explained-germanys-kurzarbeit-scheme-for-

payment-of-staff-hit-by-lost-work-hours-6329831/


Recommended